Sei sulla pagina 1di 431
Solutions to LE. Irodov’s Problems in General Physics Volume I Mechanics e Heat e Electrodynamics SECOND EDITION ABHAY KUMAR SINGH rector Abhay’ LIT. Physics Teaching Centro Patna-6 cas CBS PUBLISHERS & DISTRIBUTORS 4596/1A, 11 DARYAGAN, NEW DELHI - 110 002 (INDIA) Dedicated to my Teacher Prof. (Dr.) J. Thakur (Department of Physics, Patna University, Patna-4) ISBN :81-239-0399.5 Reprint: 2003 Copyright © Author & Publisher Al rights reserved. No part of this book may. be reproduced or {transmitted in any form or by any means, electronic or mechanical, including photocopying, recording, or any information storage and retrieval system without permission, in writing, from the publisher. Published by S.K. Jain for CBS Publishers & Distributors, 4596/1, 11 Darya Ganj, New Delhi - 110 002 (India) Printed at India Binding House, Delhi - 110 032 FOREWORD Science, in general, and physics, in particular, have evolved out of man’s quest to know beyond unknowns. Matter, radiation and their mutual interactions are basically studied in physics. Essentially, this is an experimental science. By observing appropriate phenomena in nature one arrives at a set of rules which goes to establish some basic fundamental concepts. Entire physics rests on them, Mere knowledge of them is however not enough. Ability to apply them to real day-to-day problems is required. Prof. Irodov’s book contains one such set of numerical exercises spread over a wide spectrum of physical disciplines. Some ofthe problems of the book Tong appeared to be notorious to pose serious challenges to students as well as to their teachers This book by Prof. Singh on the solutions of problems of Irodov’s book, atthe outset, seems to remove the sense of awe which at one time prevailed, Traditionally a difficult exercise to solve continues to draw the attention of concerned persons over a sufficiently long time. Once 4 logical solution for it becomes available, the difficulties associated with its solutions are forgotten very soon, This statement isnot only valid forthe solutions of simple physical problems but also to various physical phenomena ‘Nevertheless, Prof. Singh’s attempt to write a book of this magnitude deserves an all out praise. His ways of solving problems are elegant, straight forward, simple and direct. By writing this book he has definitely contributed to the cause of physics education. A word of advice to its users is however necessary. The solution to a particular problem as given in this book is never to be consulted unless an all out effort in solving it independently has been already made. Only by such judicious uses of this book one would be able to reap better benefits out of it As a teacher who has taught physics and who has been in touch with physics curricula at LLT,, Delhi for over thirty years, I earnestly feel that this book will certainly be of benefit to younger students in their formative years Dr. Dilip Kumar Roy Professor of Physics Indian Institute of Technology, Delhi ‘New Dethi-110016. FOREWORD ‘A. proper understanding of the physical laws and principles that govern nature require solutions of related problems which exemplify the principle in question and leads to a better grasp of the principles involved. It is only through experiments or through solutions ‘of multifarious problem-oriented questions can a student master the intricacies and fall ‘outs of a physical law. According to Ira M. Freeman, professor of physics of the state university of new Jersy at Rutgers and author of ‘‘physic--principles and Insights” “In certain situations mathematical formulation actually promotes intuitive understand- ing... Sometimes a mathematical formulation is not feasible, so that ordinary language must take the place of mathematics in both roles. However, Mathematics is far more rigorous and its concepts more precise than those of language. Any science that is able to make extensive use of mathematical symbolism and procedures is justly called an exact science”’. LE. Irodov’s problems in General Physics fulfills such a need. This book originally published in Russia contains about 1900 problems on mechanics, thermody- namics, molecular physics, electrodynamics, waves and oscillations, optics, atomic and nuclear physics. The book has survived the test of class room for many years as is evident from its number of reprint editions, which have appeared since the first English edition of 1981, including an Indian Edition at affordable price for Indian students Abhay Kumar Singh’s present book containing solutions to Dr. LE. Irodov's Problems in General Physics is a welcome attempt to develop a student's problem solving skills The book should be very useful for the students studying a general course in physics and also in developing their skills to answer questions normally encountered in national level entrance examinations conducted each year by various bodies for admissions to profes- sional colleges in science and technology. BP. PAL Professor of Physics ILT., Delhi PREFACE TO THE SECOND EDITION Nothing succeeds like success, they say, Now, consequent upon the warm welcome on the part of students and the teaching fraternity this revised and enlarged edition of this volume is before you. In order to make it more up-to-date and viable, a large number of problems have been streamlined with special focus ‘on the complicated and ticklish ones, 0 cater to the needs of the aspiring students. I extend my deep sense of gratitude to all those who have directly or indirectly engineered the cause of its existing status in the book world. Patna June 1997 Abhay Kumar Singh PREFACE TO THE FIRST EDITION ‘When you invisage to write a book of solutions to problems, one pertinent question crops up in the mind that—why solution! Is this to prove one's erudition? My only defence against this is that the solution is a challenge to save the scientific man hours by channelizing thoughts in a right direction. ‘The book entitled “Problems in General Physics” authored by LE. Irodov (a noted ‘and mathematician) contains 1877 intriguing problems divided into six After the acceptance of my first book “Problems in Physics”, published by Wiley Eastern Limited, I have got the courage to acknowledge the fact that good and honest ultimately win in the market place. This stimulation provided me insight to come up with my second attempt—“Solutions to ILE. Irodov’s Problems in General Physics.” This first volume encompasses solutions of first three chapters containing 1052 problems. Although a large number of problems can be solved by different methods, I have adopted standard methods and in many of the problems with helping hints for other methods. In the solutions of chapter three, the emf of a cell is represented by & (xi) in contrast to the notation used in figures and in the problem book, due to some printing difficulty. 1am thankful to my students Mr. Omprakash, Miss Neera and Miss Punam for their valuable co-operation even in my hard days while authoring the present book. I am also thankful to my younger sister Prof. Ranju Singh, my younger brother Mr. Ratan Kumar Singh, ‘my junior friend Miss Anupama Bharti, other well wishers and friends for their emotional support, At last and above all I am grateful to my Ma and Pappaji for their blessings and ‘encouragement. ABHAY KUMAR SINGH CONTENTS Foreword Preface to the second edition Preface to the first edition PART ONE PHYSICAL FUNDAMENTALS OF MECHANICS: 1.1 Kinematics 1.2. The Fundamental Equation of Dynamics 1.3 Laws of Conservation of Energy, Momemtum, and Angular Momentum 1.4 Universal Gravitation 1.5 Dynamics of a Solid Body 1.6 Elastic Deformations of a Solid Body 1.7 Hydrodynamics 1.8 Relativistic Mechanics PART TWO ‘THERMODYNAMICS AND MOLECULAR PHYSICS. 21 Equation of the Gas State. Processes 2.2 The first Law of Thermodynamics. Heat Capacity 23 Kinetic theory of Gases. Boltzmann's Law and Maxwell's Distribution 24 The Second Law of Thermodynamics. Entropy 25 Liquids. Capillary Effects 2.6 Phase Transformations 2.7. Transport Phenomena PART THREE ELECTRODYNAMICS 3.1. Constant Electric Field in Vacuum 3.2. Conductors and Dielectrics in an Electric Field 3.3. Electric Capacitance. Energy of an Electric Field 3.4 Electric Current 3.5 Constant Magnetic Field. Magnetics 3.6 Electromagnetic Induction. Maxwell's Equations 3.7 Motion of Charged Particles in Electric and Magnetic Fields 1:34 35-65 66-101 102-117 118-143, 144-155 156-167 168-183, 184-195 196-212 213-226 227-241 242-247 248-256 257-266 267-288 289-305 306-324 325-353, 354.379 380-407 408-424 PART ONE PHYSICAL FUNDAMENTALS OF MECHANICS Ll KINEMATICS 1.1 Let v, be the stream velocity and v the velocity of motorboat with respect to water. The motorboat reached point B while going downstream with velocity (y, + v?) and thea retuned with velocity (v/ ~v,) and passed the raft at point C. Let t be the time for the raft (which flows with stream with velocity v,) to move from point A to C, during which the motorboat moves from A to B and then from B to C. On svg we ge ye ge Let 5 be the total distance traversed by the point and f the time taken to cover half the distance. Further let 2: be the time to cover the rest half ofthe distance. Therefore fev ogee ® and Se eut or ae @ z Hence the sought average velocity we _ . 2 (n+¥) Rta” BF yt Dy, 1.3 As the car starts from rest and finally comes to a stop, and the rate of acceleration and deceleration are equal, the distances as well as the times taken are same in these phases of motion. Let Ar be the time for which the car moves uniformly. Then the acceleration / deceleration time is $5" each. So, 14 15 Let the particles collide at the point A (Fig 16 Hence aretVi (@ Sought average velocity 200 em 208 cre te 1 10 m/s iy, (©) For te maximum velocity, # shoud be taxinun, Fem he gue i maxima fe all points on the line ac, thus the sought ‘maximum velocity becomes average velocity for the line ac and is equal to: 0 10 2085 be 100m fe, 1G em 25 cm/s (0 Time sald be eh at careprdig we spe so pts ough he pot (rh) m tty he ety Se Fam Sue he ng pint passes through the origin and thus corresponding time f= f= 16. ), whose position vector is 73 (say) If «be the time taken by each particle to reach at point A, from triangle law of vector addition : fa re iten Heite _ A so, FoF @- Hye ® 4 ye WEA L. i erefore, = TE ® 7 Tat From Ege. (1) and @) weep ical Hen- G-77 A a RR GD 0 ® Ar R- x on, RTE = TBO wich i the sought relationship. per et $ Weave a. var, © From the vector diagram [of Eq, (1)] and using properties of triangle wy va Vibe a 2igvoos@ = 39.7km/br — @) o= sr (289) Ung @) ad puting eves ty a O- 98 Let one of the swimmer (say 1) cross the river along AB, which is obviously the shortest path, Time taken to crods the river by the swimmer 1. For the other swimmer (say 2), which follows the quickest path, the time taken to cross the river. (where AB = d is the width of the river) @ In the time ¢, drifting of the swimmer 2, becomes za rye a, (wing Ba-2) © I y be the time for swimmer 2to walk the distance x to come from C to B (Fig), then vod 2,04 Gas ose Fy (osing Ba. 3) ® ‘According to the problem f,= fy +45 dd vd wy ive , Vig On solving we get 18 Let Ibe the distance covered by the boat A along the river as well as by the boat B acr. the river. Let vp be the stream velocity and v’ the velocity of each boat with respect water. Therefore time taken by the boat A in its joumey 1 1 Tt ‘and for the boat B Hence, (On substitution tlt 18. 19 Let vp be the stream velocity and v’ the velocity of boat with respect to water. A ‘n= 2>0, some drifting of boat is inevitable. {et make an angle 0 wit ow dizecton. (Fig), then the tie taken to eros the rive rere % (where d is the width ofthe river) Vein In this time interval, the drifting of the boat x= (cos 0+¥)t d = (1 008 0+ 19) Fam (cotO +1 cose 0) d Tim Tih For fine diting) iy a caves 4 => fh (6010 +n cosec 8) = 0, which yields v! cose Le} XK” = n 2 Pd Hence, o- r 0 1.10 The solution of this problem becomes simple in the frame attached with one of the bodies. Let the body thrown straight up be 1 and the other body be 2, then for the body 1 in the frame of 2 from the kinematic equation for constant acceleration = Fae Ray Bumt Fat So, Fiz= aunt (because Wz= 0 and Fay) = 0) oe Fil Wanlt ® But [Wal= lel % So, from properties of tangle vega V ve vB 2 v9 v 0s (2/2 ~ 05) Hence, the sought distance lFl= vp V 201 -sin 6) = 22m, Lat Lu Let the velocities of the paricles (Sty 77" and V7") becomes mutually perpendicular after time f. Then their velocitis become Deeer a aeres ® ans As HLT 00, Ty's 0 or, (H+ Rt) (E+E t= 0 or =H ute 0 Hence, r= VY e Now form the Eq. Fis Faun + Haast 3 Mal? Gl+ Ils @esame nee ig 0 and Fin ™ Hence the sought distance Viale AE Veim Gslanl= 14D From the symmetry of the problem all the three points are always located at the vertices of equilateral triangles of varying side length and finally meet at the centriod of the initial ‘equilateral triangle whose side length is a, in the sought time interval (say #). % ‘Let us consider an arbitrary equilateral triangle of edge length I (say). ‘Then the rate by which 1 approaches 2, 2 approches 3, and 3 approches 1, becomes : On integrating = 13 44 Let us locate the points A and B at an arbitrary instant of time (Fig) IEA and B are separated by the distance s at this moment, then the points converge or Point A approaches B with velocity —2 = y—ucos a where angle a varies with time. On itergating, ~ fas fiv-wcos.a) a, 1 3 (where is the sought time.) r = Se scos.a) a @ ‘As both A and B cover the same distance in x-direction during the sought time interval, ‘0 the other condition which is required, can be obtained by the equation ae fue & wre frowad ® Solving (1) and (2), we get T= ea One can see that ifm v, oF u-<, plat A cannot catch B In the reference frame fxed tothe tan, the distance between the wo events is obviously eqeal to L Suppose the tain stars moving at time t= in the postive x direction and take the orgin (x= 0) atthe headlight ofthe tain at #= 0. Then the coordinate of fst vent in the earth's frame is ‘The distance between the two events is obviously. xy nage [we (t+ 1/2) = 0242 km in the reference frame fixed on the earth.. For the two events to occur at the same point in the reference frame K, moving with constant velocity V relative to the earth, the distance travelled by the frame in the time interval T must be equal to the above distance. Thus Vaml-wat+0/2) So, Vad w(t41/2) = 403 m/s The frame K must clearly be moving in a direction opposite tothe train so that if (or example) the origin of the frame coincides with the point x, on the earth at time & it ‘coincides with the point x, at time ¢+, 7 LIS (2) One good way to solve the problem is to work in the clevator’s frame having the 116 observer at its bottom (Fig.). Let us denote the separation between floor and celing by h= 2:71. and the acceleration Of the elevator by w= 1:2m/s? From the kinematical formula 1 Ym Yor Vogt +z My? @ Here y= 0,y9= +, %= and Wy = Ming) Mae) = Ca)-)= - G+") 1 2 $0, Om heh -@ew)e gew O75. (®) At the moment the bolt loses contact with the elevator, it has already aquired the velocity equal to elevator,given by Y= (12) (2)= 24m/s In the reference frame attached with the elevator shaft (Ground) and pointing the y-axis upward, we have for Pioplocemeat of he ba vt; dy= voted? f = vytehw, 2 4 1 se reseae { t Py Ay= 24) (07) +4 (-98)077= -07m, Hence the bolt comes down or displaces downward relative to the point, when it loses ‘contact with the elevator by the amount 0-7 m (Fig.). Obviously the total distance covered by the bolt during its free fall time 8 op s taylsa( o7m+ Ome 13m Let the particle 1 and 2 be at points B and A at ¢= 0 at the distances J, and J; from intersection point 0. Let us fx the inertial frame withthe particle 2. Now the particle 1 moves in relative to this reference frame witha relative velocity Vfz= vy 7, and its trajectory is the straight line BP. Obviously, the minimum distance between the particles is equal tothe length of the perpendicular AP dropped from point A on to the straight line BP (Fig.). a7 wt sy ast From Fig. (6), ve Vote, and tan = zt @ ‘The shortest distaice AP = AM sin 0 = (OA ~ OM) sin 9 = (ly 1, cot 0) sin ® v2)__¥ wh=-vh is = using 1 rae Tee me ‘The sought time can be obtained directly from the condition that (I, ~v, £7 +(l,~ va Amthy At o ar-(iek is minimum. This gives ¢ = Let the car tum off the highway ata distance x fom the point D. So, CD = x, and ifthe speed ofthe car inthe field is v, then the time taken by the car to cover the distance AC™= AD ~ x on the highway ‘AD. ny o Cx —>p and the time taken to travel the distance CB A itue Ba he a ® N So, the total time elapsed to move the car from poitAwB = SY\ | Vaz | , \ B ° LB To plot x(),5( and w,(0) let us partion the given plot v, (0) into five segments (for detailed analysis) as shown in the figure. For the part oa: w,= 1 and v= t= v : Vy 2 Ar Tans t= Suda fare Sas 4 7 ° 1" °F RBENE Ar Putting ¢= 1, we get, Axj=s= 4 4 2 7 mt For the part ab : @ vig= O and v= ve constant = 1 ‘Thus 40,0 frdi= fat= @-1)= 50) 1 Putting 103, Bey= y= Dunit For the part bd: wy= 1 and v= 1=((-3)= 4-)= 9 15 Ts 850 flt-tn 8-3 gi Lins Putting t= 4, Bay= y= Sanit For the part 4d: v= -Land v= ~(1-4)= 4-1 So, 1-4 for 04 ‘Ts Anis fd-ode ar Putting f= 6 Axe Similarly 560 fivgldem fe-aydr= Putting t= 6, sy unit For the patd7: — w,= 2 and v= ~242(¢-6)= 2(¢-7) ve [yl= 20-8) for 1-7 Now, Ax ()= [2 ¢- dim P1404 48 Pating te 4 Atge 1 Siitaty std f20 dew 14-248 Putting fT sged On the basis of these obtained expressions w, (), x(t) and s (0) plots can be easily plotted as shown in the figure of answersheet. 10 1s 1.20 (© Mean velocity Total distance covered Time elapsed = £2 2B soem a) Vo (©) Modulus of mean velocity vector i t [<3 I+ lal, 2B. ems @ (©) Let the point moves from i wo f along the half circle (Fig) and vp and vbe the spe athe points respectively. a We have Hew, 0% Vm vot wt (as Ww, is constant, according to the problem) Jf Oo+mode emp tot (op + 2) $0, ope ¢ Se So, from (1) and (3) vot 2 e Now the modulus of the mean vector of total acceleration 6 ~ [cite fe 28K | Ine (@) we have rd So, and (©) From the equation So, te sought tine ar= 2 As Ve @-200 a(.-2a0) forrs 1 a(2ar-1) fore> x5 121 ua Hence, the sought distance i20 ve tr fea f aG-2egds f e@arne ‘Simplifying, we get, s= 37 (@) As the particle leaves the origin at r= 0 So, are ref w As na So, ¥* % ( - @ From (1) and (2), a Sof-Je- vo) 6 Hence x coordinate of the particle at ¢= 6s. ms rox6(t-s23)- 24em= 024m 25 Sint on ‘0 = x(t 32) ant ems 20 x sox20(1-35)- = 200 em = = 2m (b) At the moments the particle is at a distance of 10 cm from the origin, x = = 10cm. Putting xe +10 in Eq. 3) 1o= s(t } o, 7-104 10= 0, $0, tne EOD 52 vs Now potting x= ~10in Eqn (3) -10 10(1-75}: On solving, te 52 VB os As £ cannot be negative, so, 12(5+V35)s 12 1a2z Hence the particle is at a distance of 10 cm from the origin at three moments of time : t= S2Vid 5, S4VB 5 Q (©) We have a af1—4 ~ vane tds nofe-alf o> G So se Malt 3) tor te x= vert -re9 ates aed ve fnfi-afofa rere = vel + (=) 12 for #> ® sof odes fae eco And for t= 88 : B so fof-garfrofe-s)a 0 inept a impinge ge $= 34em. (On the basis of Eqs. (3) and (4), x (¢) and s(t) plots can be drawn as shown in the answer sheet As particle is in unidirectional motion it is directed along the x-axis all the time. As at t= 0,x=0 a So, Axa x= s,and = w Therefore, vm avE= ave o As, On integrating, @ 123 13. (©) Let s be the time to cover first s m of the path. From the Eq, sa fea se feato coms Ns ® ‘The mean velocity of particle 2V5/0 fu _feoa | 0 ae fa 2Vs/a 2 According to the problem véy : =A. ar (a v detest with tie) o -f Warm ofas % ° Om integrating we get 5 = Zu ‘Again according to the problem ay av Spo ae or a a 9 av o, Le ofa Thus @ as Fatitory So, xm at, y= be? bx? and therefore ye bee 14 125 which is Eq. of a parabola, whose graph is shown in the Fig. (b) As Fe atitbe2j" Ee arte @ So, v= Va7(=2b1) = Vata be itt. Eq, (1) wort, time, we get ca a - 27" So, [P]-wa2b © coon FW, (T2617) (- 207) vw” WaFe abr) 2 or, cos. = 2 bt . Vata” 0, unas xt (@) The mean velocity vector oT ait oj Hence, l |= VaTe (=b 0" = VaTS BPE (@) We have a) Hence, y (x) becomes, (©) Diuterentinting Ba. (1) we get v= @ and ¥,= @(1-2a2) @ 126 127 18, Dit. Ba. (2) with respect to time m2 04nd W,*-200 0, we Vala i 20a (©) From Eqs. @) and (9) Wehave Th afha(1-2ar)j* md We 2oaj* So, cos Z= 1. EH -a(1-2a)2aa 4° V2" yw" aVI+ (2am) aa On simplifying. 1-2ate 21 As a0, ged Differentiating motion law : x= asin@r, y= a(1-cos@s), with respect to time, v= @OOsar, v= asin or So, Fe aces orjt aosinarj” o and v= aos Const ® Differentiating Eq. (1) with respect to time o.oo. tw Trew?ecer” 07 sin ati 207 cos at] ® (@) The distance # waversed by the point during the time + i given by s=Jvam fawd= aor (using2) of (©) Taking inner product of 7” and WF We get, P= (a ocos ott aesinar]} (a0? sinor(-i)+ a0? cos ar -j5 So, Pte ~aPu*sinar os wt + (2) te anticlockwise sense of angulr displacement a8 positive. ‘Also from the geometry of the triangle OAC R sin” sin(x-26) Let us waite, F% reos O74 rsinOj® 2Roos?O]% Rsin20j7 Differentiating with respect to time. or, r= 2Reos 8. © om ino) 224 dos © op Fe 2R2cos0 (—sind) 22 74.2 R cas 20407 st on (42) (sin 2072 cos2077 0, 2a(=g2) (sin207 eos20j] 01, 7 2Re(sin2 01 cos? Oj) So, [¥] or v= 20R=0-4m/s. : : a, ‘As o is constant, vis also constant and w,~ 4” = 0, 2 2 2, GORY 429-032 m/e? So, we mye a GORY. 4o2R= 032 m/s Aber: om Fhe ng aero Cte (28) do ee ‘Thus we have the problem of finding the velocity and acceleration of a particle moving along a circle of radius R with constant angular velocity 20. of the point A, with respect to centre of }-20= constant at Hence v= 20R and vy? (2oRP a wee ee CORY gate R Differentiating @ (+) with respect to time dg Gen Orn 2at a For fixed axis rotation, the speed of the point A: vaoR= 201k or Rx 5*- @ 26 145 1.46 Differentiating with respect to time yn Bo 20k», (using 1) ‘The shell acquires a constant angular acceleration atthe same time as it accelerates linearly. ‘The two are related by (assuming both are constant) eas 1” Zan Where w= linear acceleration and = angular acceleration Then, o- V2p2an~ V 2% (220F But v= 2w/, hence finally ou 2E0Y Let us take the rotation axis as z-axis whose positive direction is associated with the Positive dsection of the cordinate g, the rotation ange, in accordance with the right-hand screw rule (Fig.) (@) Detferentating (+) with respect to time. z do 2 Se a-3017- 0, @) and #9 do, dit dt ~6br @ From (1) the solid comes to stop at Ar= r= 3bt?, for 0stsVa/3b ‘The angular velocity w = (a-3bP)dt Sar “Pe So, <@>= = [arb de Similarly B= |B, 654 for all values of ¢. a7 Var 6be de So, = eae, = V3ab Sat “Pi (&) From Eq. @) B,~ -6b¢ So, (8) .= a7 = 06 VE = -2Vab Hence Bx |(B),. yarap [7 258 Angle cis related with |w,] and w, by means of the fomula : tmnam 7, whee Wye OPK ad [l= BR ® where is the ada ofthe circ which an abizarypoit ofthe body ccumserbes From the given equation p= 22~ or ire = £2» as Bis postive for ll values of) hanging win eink Fn fds or, whe? ir & ween) a and Iwl= BR» atk Putting the values of | ,| and w, in Eq. (1), we get, @s'R/4 at? 4 i want 28, 1a SJome] In accordance with the problem, B, < 0 Thus -42.= & VS, where k is proportionality constant t or, - Terk fae of, Vo = Vag ~ a) 2 E When c= 0, total time of rotation ¢= <= 149 150 2Veag/k fod f (oF note) ae far" Volk Average angular velocity < @ > seme >= [ate Bes af Wetwe wnay-ag = # Inert thi Eq within‘ iit fo (9) Heace oe Bae") ® (0 Fiom the By, = oye and Ee. or by ecntating Ea (1) om aye Lgt us choose the positive direction of z-axis (stationary rotati B,, In accordance with the equation a do, Gin Be oF 0 FB, mn axis) along the vector 0, 0, da, B,d9= Boos ede, Integeating this Eq. within its 1 ,@) feo. tof os ede ° : wee ising Hence = = V2Bysing ‘The plot w, (@) is shown in the Fig, t can be seen that asthe angle g grows, the vector Test increases, coinciding with the direction ofthe vector Bi (o,> 0), eaches the maximum stg» @/2 then sas decreasing and finally tus into zero at @ = x. After thatthe body Starts rotating in the opposite direction ina similar fashion (o, <0). Asa result, the body will oscilate about the postion @= 9/2 with an amplitude equal to 2/2 1.51 Rotating disc moves slong the x-axis, in plane motion in xy plane, Plane motion of a soli known a instantaneous centre of rotation. The instantaneous axis whose positive sense is directed slong ao he slid and which pasts through the points known a instantaneous axis of rotation. ‘Therefore the velocity vector ofan arbitrary point (P) ofthe sli canbe represented 8 can be imagined to be in pure rotation about a point (say J) at a certain instant yo Sip a On the basis of Eq, (1) for the C. M. (C) of the dise ¥ Wee Bury @ According to the problem itt i and Bt ie, TLx-y plane, so t0 satisy the Eqn. (2)7s ditected along (~]'). Hence point is ata distance roy y, above the centre of the dise slong y-axis. Using all these facts in Bq. @), we get x ven ay or y= “E ® (©) From the angular kinematical equation = +844 © wn Br (On the other hand x= vt, (where x is the x coordinate of the CM.) o, me © From Ege. (4) and (5), © = : v v2 Using is ae ot i By @) we pte Bee gt ee Zappa) (b) As centre C moves with constant acceleration , with zero initial velocity So, we? and v= we 7 z Therefore, v0 WRF «view Hence yn tea VEE (parabola) 30 1sz 153 ‘The plane motion of a solid cam be imagined as the combination of translation of the CM. and rotation about CM. So, we may write y= ¥o+%o =. aileaer, — ()and 4 An Retire yey | 7 aio 40? (~The) +B) 2) | 2 Fa is the position of veetor of A with respect to C+ In the problem vo= v= constant, and the rolling is without slipping ie, vow v= oR, So, We 0 and = 0, Using these conditions in Eq, (2) War 0? (=Tac)™ OP R(~tye)= F(~iye) Here, iy ¢ is the unit vector directed along 7c Hence w,~ "5 and w, is directed along (~ 4, ¢) or directed toward the centre of the wheel. (©) Let the centre of the whee! move toward right (positive x-axis) then for pure tolling ‘0m the rigid horizontal surface, wheel will have to rotate in clockwise sense. If be the angular velocity of the wheel then w= 3 ee Let the point A touches the horizontal surface at £= 0, further let us locate the point A ares ‘When it makes @= «at the centre of the wheel. From Eqn.(1) f= WE+Tx roe = v5 0 CB) x [Roos 0 (JF +R sin 0(- iJ} or, Tha vIFOR[cosor-i)+sinotj ] = (v~ 008 wt) 74 v sin wtj” (as v= aR) V (v= v008 on)? + (vsin ot)? = vV 2(1-cos wr) = 2 vsin (wt /2) Hence distance covered by the point A during T= 2x/e so fraee f rvsinoerya- © : Let us fix the co-ordinate axis xyz as shown in the fig. As the ball rolls without slipping. along the rigid surface so, on the basis of the solution of problem 1.52 : aR. Be eaxrZs 0 uo ve oR and tt (Ck) as ena} o H+ Pur=0 4 BR and Btt CE) as tt A_VetWR=Va ‘At the position comesponding to that of Fig, in accordance with the problem, 8 arate state j \ and ® % * J) % (2) Let us fix the co-ordinate system with the frame 777777 attached with the rigid surface as shown in the Fig. ‘As point O is the instantaneous centre of rotation of the ball at the moment shown in Fig. 0, io= 0, Now, z = satire a uFha Bier Ge e+ oR) So, Ux 2vcim Zwei using 1) Similalry y= T+Tx Fyo= volt o(-BxR GO is So, vg=VEv,=VE wt and ¥y is at an angle 45° from both Find j (Fig) ©) He Wet 0? FR) + Bie w veThaR ie vei hel % ea AW $6 = O70) Clee) (using 1) where lige is the unit vector along Fe 3 we awe an 10, mye BE wsing 2) and i diced wars te cent ofthe tll Nowe Rene Cre Pihe it = wid oP Rj} + BR) XRF ats aes = or RYE CT (ose 1) = a7 EC] So, man V set tt «aw V 14] Similarly Wy = + 0 (— 750) + BX Fac = wi oR C+ CHRO) Bae ot - (eB) ing » : = (meine 9 So, 1.54 Let us draw the kinematical diagram of the rolling cylinder on the basis of the solution of problem 1.53. Wet Br AAs, an arbitrary point of the cylinder follows a curve, its normal acceleration and radius of curvature are related by the well known equation Z R vA 0, for point A, wane Similarly for point B, vy o*, Ryn NEE n ar 1.55 The angular voc isa vector as inaesinal roe commu. Thea he reatve agua elo of te body 1 with ect he body 2 sea Bye ss frre tea voc. The mate seein of wa. 2 156 usr 33 where 5" is a frame corotating with the second body and S is a space fixed frame with origin coinciding with the point of intersection of the two axes, oo (ea a “a a . — &, Sine 5" routes with angular Vly However () « os ett body routes ‘with constant angular velocity in space, thus Bam @ xa, Note that for any vector By the relation in space forced frame (&) and a frame (X’) rotating with angular velocity ais a5) dB) oe “4 +r cle Webave Ge atid be?j” @ dt So, om VGH OF, thus, o,. gy. 781 rad/s Differentiating Eg. (1) with respect to time Be 48. atom” @ 50, p-Vereny and Ble wos * 13 rad/s? ® cose SB, (ati be 2bij) OB” Yan? Ory Va + (bye Putting the values f(a) and (b) andtaking #~ 10s, we get ai? (@ Let the axis of the cone (OC) rotates in anticlockwise sense with constant angular velocity @ and the cone itself about i's own axis (OC) in clockwise sense with angular ‘velocity (Fig). Then the resultant angular velocity ofthe cone. wT sa, ® As the rolling is pure the magnitudes of the veclor & and can be easly found from Fig Reota’ %" ”R ® Asa Lah trom Eq, (1) and @) 34 18s on Vad Ges) -eeae (b) Vector of angular acceleratic dv_4@+H) ria a (as @ = constant.) ‘The vector i which rotates about the OO’ axis with the angular velocity &, retains i ‘magnitude, This increment in the time interval dt is equal to [dB l= oy 0" dt or in vector form d= (Wx ) de Thus Be axa ‘The magnitude of the vector Bis equal to Be wag (as T 15) SB Rear” GEE 23 nd/s The axis AB acquired the angular velocity wR ® 5 Using the facts of the solution of 1.57, the > o> ngulae velocity of the body O74 won Vorvo™ = Vohe fie = 06 rd/s ‘And the angular acceleration. pe dd AO) aa dy eae aE But so So, Be (Bhex a) +B AS, PpLay 50, B= VeopBol +e = BVI + (mpi = 02 rad/s? 2 9 0 38 ‘THE FUNDAMENTAL EQUATION OF DYNAMICS Let R be the constant upward thurst on the serostat of mass mt, coming down with a constant acceleration w. Applying Newton's second law of motion forthe aerostat in projection form R= mw, mg-R= mw ® Now, if Am be the mass, to be dumped, then using the Eq. F,= mw, R-(m- Am) g= (m= Am), @ From Eqs. (1) and (2), we get, Let us write the fundamental equation of dynamics for all the three blocks in terms of projections, having taken the positive direction of x and y axes as shown in Fig; and using ‘the fact that kinematical relation between the accelerations is such that the blocks move with same value of acceleration (say ) m8-T,= myw @ x AN N2 T,-T,- kn, g=m,w @ x ; BR ad Ty-kmg= ma ) ff ir in The simultaneous solution of Eqs. (1), (2) and mn ©) yields, q mg ng [mp km, +m) ] | mora «oo (+k) 1 Tye Seb nd A” gem, ree ‘mog [As the block m moves down with acceleration w, so in vector form ape tox km + ma 1B" ‘mig + y+ Let us indicate the positive direction of x-axis along the incline (Fig). Figures show the force diagram for the blocks. Let, R be the force of interaction between the bars and they are obviously sliding down ‘with the same constant acceleration w. 182 ‘Newton's second law of motion in projection form along x-axis for the blocks gives : m, gsin a —k,m, gcosa+R= mw @ smygsin ot R— ky g 008 = mw @ Solving Eqs. (1) and (2) simultaneously, we get m+ amthm oy im, +p mm (y= by) g008 0 mm (©) when the blocks jut side down the plane, w= 0, so from Eqn. (3) kmthm mm on, (m+ m)sin am (&,m;+kjm;) cos a _ emt ham) mm (Case 1. When the body is launched up : Let k be the coefficeint of friction, u the velocity of projection and I the distance traversed slong the incline. Retarding force on the Block = mg sin a-+kmg cosa. and hence the retardation = gina + kg cos a. Using the equation of particle kinematics along the incline, O= -2gsina+kg cosa)! ce ® we gsina-g.cos a Re ® gsin a - gos a. ° Hence tana Go TEsina+kgew a) and O= u-(gsina+kgcosa)t or, um (sina +kgcos.a)s @ Using @) in () = Lgsina + egeas oe? ® Case 2). When the block comes downward, the net frce onthe body mg sn c.— fo gco8 cand hence its acceleration = gin c~ cos Let tbe the time required then, I= Eigsina—kg cosa)? o From Eqs. (3) and (4) (according to the question), Hence on solving we get 37 1463 At the initial moment, obviously the tension in the thread connecting m, and m, equals the weight of my (@) For the block m, to come down or the block m, to go up, the conditions is mg-T20 and T-m,gsina-fr20 ‘where T is tension and f, is friction which in the limiting case equals km,g cosa. Then orm, g—m,sina> km, gcos a o 72 cosa + sina) Not (©) Similarly in the case i 1m 88in — m, 8> Fig ie i on, m, gsin am, g> km, gc0s 0 “tig mg o, 72 < (sin kos a) (©) For this case, neither kind of motion is possible, and fr need not be limiting, Hence, (cos a + sin a) >"> (sin a - kos @) m 1L64 From the conditions, obtained in the previous problem, first we will check whether the mass m, goes up or down, Here, ma/m, = 11> sin a + kos a, (substituting the values). Hence the mass m will come an acceleration (say w). From th free body diagram of previous problem, mog-T- mw © and Tm, gsin a km, go0s.a= mw ® ‘Adding (1) and_(2), we get, my g—m, gsin km, gos (m, +m) w won 22 ~Sin a — Koos 4) 8 (n— sina — kos a) g (+ m/m) 17 Substituting al the values, w= 09048 g ~005 g ‘As m, moves down with acceleration of magnitude w= 0.05 g >0, thus in vector form acceleration of m= sp a (=sin a kos a) 2” he Ten oos gt 165 Let us write the Newton's second law in projection form along positive x-axis for the plank and the bar frm my wy, frm mg wy @ 1.6 1.67 ‘At the initial moment, fr represents the static friction, and as the force F grows so does the friction force fr, but up to it’s limiting value 4. fr™ guy EN= km. fr_[mpor ‘Unless this valu is reached, both bodies moves.“ / 7 > fr 4s a single body with equal acceleration. But im soon as the force fr reaches the limit, the 7/7//M™TTT/MMM bar stars sliding over the plank Le. w,2 Ww). ‘Substituting here the values of w, and wy taken from Eq. (1) and taking into account that fp= km, g,we obtain, (at - km, g)/m,= 2 ere thesign * corresponds to the moment t= fo (say) gm, (+m) 1 tty then w, = “228 const) and Ma (@t~ kang g)/m, On this basis w, (2) and w(t), plots are as shown in the figure of answersheet. Let us designate the x-axis (Fig.) and apply F,= mw, for body A: mg sin a- km gcos a= mw or, we gsina-kgoosa Now, from kinematical equation : Ise a= 0+ (1/2) ws? or, t= ViTsec a/(in a - keos a) g N = Vill Ginda/2— eos ag vy (using Eq. (1)). for tae, k => te 2828.52 pcosasina= 0 b or, a= 49° tun2a= and putting the values of a, k and ! in Eq. (2) We get fie Let us fix the x—y co-ordinate system to the wedge, taking the x - axis up, slong the incline and the y— axis perpendicular to it (Fig.). 168 Now, we draw the free body diagram for the bar. Letus apply Newton's second law in projection form along x and y axis for the bar : Tcos B-mgsina-fr= 0 a TsinB+N-mgcosa= 0 or, N= mgcosa-TsinB @ But f= AV and using (2) in (1), we get T= mgsin.a + kmg cos a/(cos 6+ ksin 8) ® For Trg the value of (cos B+ ksin B) should be maximum Putting this value of B in Eq. (3) we get, 7. = —mplsina + keosa) __ mg ina+ kcosa) meV +e svi View First of all let us draw the free body diagram for the small body of mass m and indicate x-axis along the horizontal plane and y - axis, perpendicular to it, as show in the figure. Let the block breaks off the plane at f= fp ic. N= 0 So, N= mg-atsina= 0 oye TE a 4 6 0" Fina ® 5 From F, = mw,, for the body under investigation TTT nd yt atcos a ; Integrating within the limits for v (0) mf dy, cosa fae (using Eq. 1) ° So, ve @ Integrating, Eqn. (2) for s (0) acosas? oT @) Using the value of ¢= f from Eq. (1), into Eqs. (2) and (3) yan MEO sag 5, Mgrcose 2asin'a 60° sin a 40 1069 1.70 1m Newton’s second law of motion in projection form, slong horizontal or x—axis ic. F,= mw, gives. a Foos(as)= mv (as a as) ot, Fees (as) ds = mvdv Integrating, over the limits for v(s) Efe (as) ds ye VEE mg = Vigsina/3a (using F= re) is the sought relationship. From the Newton’s second law in projection from : For the bar, T~2kmg = (2m)w @) For the motor, T-long= mw ® Now, from the equation of kinematics in he frame of bar oF motor : Ie Lovey? ® From (1), @2) and (3) we get on eliminating 7 and w/ t= V2I/(kg +3) r rz, 2m -t > fr fe Let us write Newton's second law in vector ftom F"= mi, for both the blocks (inthe frame of ground). Toma mi @ TemR mi, @ These two equations cogsin three unknown quantities i,» and 7. The third equation is provided by the kinematic relationship Delncen he aceleron h et, hm) where i” is th acceleration ofthe mass my wth respect tothe pulley or levator ca Summing up termoise the lef band and the righthand sides of these kinematical equations, we get a Writs 2% @ The simultaneous solution of Eqs-(1), (2) and (4) yields ap = (mz) 8+ 2m, iy : my, Using this result in Eq, (3), we get, we MIM oy z Fe EG i) and T= TOT im, 2m mz» 8) Using the results in Eq. @) we get i = ei) mm: (©) obviously the force exerted by the pulley on the celing of the car Am ms mm Fm) Note : one could also solve this problem in the frame of elevator car. Fe -27- Let us write Newton’s second law for both, bar 1 and body 2 in terms of projection having taken the positive direction of x, and.x, as shown in the figure and assuming that body 2 starts sliding, say, upward along the incline T,~m, gsina= mw, @ y at m,g~T,= mw @ t For the pulley, moving in vertical direction Ti from the equation F,= mw, 27,-T,= (m,)w, tg. bh | (2s mass of the pulley m,= 0) y2 V2 or T,=2T, ) T ‘As the length of the threads are constant, the ‘mg. jonship of accelerations we 2m o ‘Simultaneous solutions of all these equations yields : ) 2g(21~sina) (n+1) 42 473 474 Let us write Newton's second law for masses m, and m, and moving pully in vertical direction along positive x-axis (Fig.) : mg-To mW, OW | er To mW m8 m, @ Me i T,-2T= 0(asm= 0) or -2r @ Ti Again using Newton's second law in projection form for mass my along positive x, direction 7 |e Gig), we get T Tyo mgm @ The kinematical relationship between the t 1 |» accelerations of masses gives in terms of ™ projesion onthe == a ¢ Ym2g te 2m Simultaneous solution of the obtained five equations yields: __ Edm m+ mg (m= me) 1 mm mm +m) In vector form ag _ Hmima + mlm =m) 1B 1” Bm my + mg (m+ 3) AS the thread is not tied with m, s0 if there were no friction between the thread and the ball, the tension inthe tread would be zero and a a result both bode will have fee fall motion. Obviously inthe given problem iis the fiction force exerted by the ballon the thread, which becomes the tension inthe trea, From the condition of language of the problem wyy> Wy and as both are directed downward 40, relative acceleration of M = vy~™q and is directed downward, Kinemtical equation for the ballin the frame of rod in projection form slong upward direction gives: Fe Roy)? ® (hy [Newton's second law in projection form slong Tefp vertically down direction for both, rod and ball 5 tres, Mg frm Mity ® fr ng fr= mig ® ‘el nb ym Multiplying Eq, @) by m and Ea, (3) by ME | fg and then subtracting Eq) from (2) and afer sing By. (1) we get 1 21Mm © Mme "4 43 175. Suppose, the ball goes up with aceleration w, and the rod comes down withthe acceleration Wy. As the length of the thread is constant, 2, = w, @ From Newton’s second law in projection form along vertically upward for the ball and vertically downward for the rod respectively gives, T-mg= mw, @ and Mg-T'= Mw, @ but T=2T (because pulley is massless) @ From Eqs. (1), (2), (9) and (4) vy, - Mame mms 1 mea ed and yu 20=DE 7" (+4) From kinematical equation in projection form, we get 1 I (an upward direction) (downwards) (wy +e? z as, w, and w, are in the opposite direction. Putting the values of w, and w, the sought time becomes t= V21(q4+4)/3(2-ng = 14s 1.76 Using Newton’s second law in projection form along x-axis for the body 1 and along negative x ~ axis forthe body 2 respectively, we get mg-T.= mm, ® Th-m,g= mM, @ For the pulley lowering in downward direction from Newtons law along x T,-2T,= 0 (as pulley is mass less) or, T,= 27, @ As the length of the thread is constant so, w= 2m, Oy ‘The simultancous solution of above equations yields, 2(m,-2m)8 2-2) 4. ™ Taste ag Os Fhe © Obviously during the time interval in which the body 1 comes to the horizontal floor ‘covering the distance f, the body 2 moves upward the distance 2h, At the moment when the body 2 is at the height 2h from the floor its velocity is given by the expression : sie egcnn=2| DE S nee as a ca ea ean tees ae mee ene ed eet naar teh ea neta te oe ee Ln 178 ‘Owing to the velocity v, at that moment or at the height 2h from the floor, the body 2 further goes up under gravity by the distance, po, sian a ont4 ‘Thus the sought maximum height attained by the body 2 : Ha thei = 24+ MO=2, Sah +4) ned Let us draw free body diagram of each body, ic. of rod A and of wedge B and also draw the kinemetical diagram for accelerations, afier analysing the directions of motion of ‘Aaand B. Kinematical relationship of accelarations is: tuna “4 @ Let us write Newton's second law for both bodies in terms of projections having taken positive directions of y and x axes as shown in the figure. eee N cee es ® and Nsina= mgwy @) ‘Simultaneous sotution of (1), (2) and (3) yields = my 8 sin ™ -—f£__ ,, atm, cotacea” Gsncoa “ana” (ana+neota) ha if s, 44 Zeng Ws Note : We may also solve ths problem using conservation of mechanical energy instead of Newton’ second law. Let us draw fe body diagram of each body and fix the coordinate system, as shown in the figure. After analysing the motion of M and m on the basis of force diagrams, let us draw the kinematical diagram for accelerations (Fig)- As the length of threads are constant so, spy diy and a8 Vay and Vig do not change their directions that why [ae | = he] = 7 Ga and Daag tt ing and hy th hy 179 ais r rT fs ' -N Ny hy ttn 7 iy le r a wk Asai tty to, fiom the tiangle law of vector addition wan vow ® From the Eq, Fy= my forthe wedge and block: T-N= Mw, 2 sod N= me ® Now, from the Eq, F,= mw, forthe Block mg-T-KN= mov ® ‘Simultaneous solution of Eqs. (2), (3) and (4) yield we Me Gan m+) ~ (E+ 2+ Mm) Hence using Eq. (1) =e _ a" Gy ks Mim) Bodies 1 and 2 will remain at rest with repect to bar A for Wyig < WS Wag, WHETE Wi the sought minimum acceleration of the bar. Beyond these limits there will be a relative motion between bar and the bodies. For 0. k 1.81 Let us draw the force diagram of each body, ang on this basis we observe that the prism ‘moves towards right say with an acceler ion w and the bat 2 of mass m, moves down the plane with respect to 1, say Wy, then, w= wy, +; Fig.) Let us write Newion's second law for both bodies in projection form along positive yy and xy axes a5 shown inthe Fig. mos No may." Ma[ Wain) * Mig) ]* Mal Ot Msin a] o, m,ge0s a-N= mW, sina ® and Nsina = my, @ Solving (1) and (2), we get ye Mngsinacosa __gsina cosa 1m, +m, sin? a (1m,/m,) + sin? a N ey Ar Ee; DZ Wr a7 1.82 To analyse the kinematic relations between the bodies, sketch the force diagram of each body as shown inthe figure. On the basis of force diagram, it is obvious thatthe wedge M will move towards right and the block will move dow along the wedge. As the length of the thread is constant, the distance travelled by the block on the wedge must be equal t the distance travelled by the wedge on the floor. Hence d5yy = dye As Voy and vz, do not change their directions and acceleration that’s why yy tt Vag and Why tt Why aNd yyy Wag (say) and accordingly the diagram of kinematical dependence is shown in figure. AS Wig Wag + Whe $0 from triangle law of vector addition. mm Wma + el Wyn Vag twa 2 Wm wy cose = wVHT 008 a) ® From F,= mw,, (for the wedge), T= Tcosa+Nsina= Mw @ For the bar m let us fix (x -y) coordinate system in the frame of ground Newton's law in projection form along x and y axes (Fig) gives img sina ~ T= Wr [Haart Mat] =m [Mat mec08 a) =mw (tensa) GB) mgcona-N= mig) mL Muay) Mars) ] mCO+ WSN} ® Solving the above Eqs. simultaneously, we get we masing _ My2m(1- cosa) Note : We can study the motion of the block m in the frame of wedge also, alternately ‘we may solve this problem using conservation of mechanical energy. 1.83. Let us sketch the diagram for the motion of the particle of mass m along the circle of radius R and indicate x and y axis, as shown in the figure. (@) For the particle, change in momentum 4 p= mv (-i)-mv(@) so, [Ap |= V2mv and time taken in describing quarter of the circle, 184 1.85 aR ane Fp ew LPL, Ew | WE mv? a Hence, = "Ke" xR/ty” aR ye = () In this case f VE P= 0 and py= mw,t(-i, so [Ap l= mmc td Fs) - 14a Hence, || = 1PL= mw, While moving ina loop, nomal reaction exerted by the fier on the loop at diferent points and uncompensated weight if any contribute tothe weight of ier at thse points. (@) When te aircrafts atthe lowermost point, Newton's second law of motion in projection form F, = mm, ves N-mg= ™ i" m4 my? N or, N= mg +22 = 2.09 KN wt (©) When itis atthe upper most point, again fiom F,= mv, we get mg, me Ns mg = ene mg Ne Omg e OTEN (©) When the aircraft is at the middle point of the loop, again from F, = mw, 2 Na Be 4 The uncompensated weight is mg. Thus effective weight = VI mF =1-56KN acts obliquely. Let us depict the forces acting on the small sphere m, (at an arbitrary position when the thread makes an angle 0 from the vertical) and write equation F'= m #via projection on the unit vectors (and iy From F,= mw,, we have in =m mg sin 0 =m: a mee pe ds "T-d0) (as vertical is refrence line of angular position) 49 or vdv= ~glsin0d0 Integrating botb the sides : Ae var pif snoao freon or, Fa glooso Hence "+ 2g cos 8 , @ (Ea. (1) cam be easly obtained by the conservation of mechanical ene)) Fon Fy» mw, my? T-mg.coso=™ Using (1) we have T= 3mg.cos 0 @ ‘Again from the Eq. F,= mw, img sinO= mw, ot w= gsind ® Hence w= Vw? +w2 = V(gsin0)*+ (2g cos 6) (using 1 and 3) = gVie3 e076 (©) Vertical component of velocity, v,= vsin 8 So, vz v?sin?@= 2g /cos 6 sin76 (using 1) For maximum which yields Therefore from (2) T= 3mg Fem VE mg (0) We bave We 05,40, 8, us oy, Wy) My) But in accordance with the problem w, = 0 So, Wat Way) 0 or, gin O sin 0 + 2g cos 70 (~cos 0) = 0 «. c= gy oe 80 547 50 1.86 187 ‘The ball has only normal acceleration at the lowest position and only tangential acceleration at any of the extreme position. Let v be the speed of the ball at its lowest position and bbe the length of the thread, then according to the problem 2 Se gsina — () where otis the maximum deflection angle From Newton's law in projection form : F,= mw, = mg sin = mv A or — ~glsinOd0~= vdv (On inerating bot the sides within tet Him, nf meae- Soa o, = 2g! (1-co8 @) ® Note : Eq. @2) can easily be obtained by the conservation of mechanical energy of the tall in the uniform Geld of gravity. From Fas. (1) and (2) with @ = a 2gl 1 cos a)» Ig cos 5 quae? aes Lets dep he ges tng ont body (ve ae he oreo gst niin he rma eaion JP) and wre equnion P5'mivin precio ee wi eco Pint i) From F,= mw, dv masno= nt mitt 2d eo a6 or, gRsinOd0= vay Integrating both side for obtaining v (0) Sensnoaoe oar oF, v= 2gR (1 00s 0) mgcos 0-N= mi 2 ‘At the moment the body loses contact with the surface, N= 0 and therefore the Eq. (2) becomes we gh cos 0 ® 51 Where vand 8 correspond to the moment when the body loses contact with the surface Solving Eqs. (1) and (3) we obtain cos 0 = g of, O= cos”! (2/3) and v= VIGR7S 188 At frst draw the free body diagram of the device as, shown, The foree, acting on the sleeve are it's weight, acting vertically downward, spring force, along the length of the spring and normal reaction by the rod, perpendicular to its length. Let F be the spring force, and Al be the elongation, From, F,= mv, : NsinO + F cos 0= mar o where 760s 8 = (Ip + AD. Similarly from F,= mw, Neos0-Fsin@= 0 of, N=Fsin O/cos 0 @ From (1) and @) F (sin 0/cos 6) -sin 0+ F cos 8 = mar = mo? (y+ Al)/cos 6 On putting F= «Al, K Alsin? 0+ Alcos?0= ma*(j+ 41) on solving, we get, 1% ok K-mo (x/ma*-1) and it is independent of the direction of rotation. 189 According to the question, the cyclist moves along the circular path and the centripetal Al= mot force is provided by the fictional force. Thus from the equation F, = mo, r ¥ or b{t-g)ee Sot vm kg(r-F/R)g a q For Vgayy We should have or 2r 1-Ze 0, so r= R72 Hencs vue # EV Toa 1.90 AS initial velocity is zer0 thus v= ws @ ‘As w,>0 the speed of the car increases with time or distance. Til the moment, siding sta, the state friton provides the required centipeal acceleration tothe cat, Ths fre mo, but frs kg 191 192 se, Wake on veceke or, vse -wR Hence Ym VOPR = ; Yow 1/7 to, trom Eqn (1), the sought distance = 2 BY 1 = com. Since theca follows a curve, so the maximum velocity a which ican ride without sliding at the point of minimum radius of curvature is the sought velocity and obviously in this ‘case the static friction between the car and the road is limiting. Hence from the equation F,, ding = on va VERE 80, Vaan = VERuin 8 « a) ‘We know that, radius of curvature for » curve at any point (x, y) is given as, 1+ (dy/aep @ @y la? For the given curve, > See () a #y. a a ae Substituting this value in (2) we get, at + (@/e2) 00s? (x/a) P? (a/e?) sin (x/a) Fo the minimom R, and therefore, corresponding radius of curvature Reig ® Hence from (1) and (2) Vue = OV TG7E ‘The sought tensile stress acts on each element of the chain, Hence divide the chain into ‘small, similar elements 50 that each element may be assumed as a particle. We consider ‘one such clement of mass dm, which subtends angle dt at the centre. The chain moves along a circle of known radius R with a known angular speed « and certain forces act on it We have to find one of these forces. From Newton's second law in projection form, F, = mw, we get 2 sin (da/2) — dN cos 0 = dma? R and from Fz= mw, we get aN sind = gdm Then putting dm = mda/2-x and sin (da/2)= da/2 and solving, we get, _ Mo" R +g cot 0) r 2 Yoo 193 Let, us considera small element of the thread and draw free body diagram for this element. (@) ‘Applying Newton's second law of motion in projection form, F, » mv, for this element, (T+ 47) sin (d 0/2) + Tsin (4 0/2)-dN= dmw?R= 0 or, -2F'sin (40/2) = AN, (negelecting the term(d7 sin d 8/2) ] or, Td0~ aN, as sin 40. 20 @ Also, d fre kdN= (T+dT)-To dT t @ From Eqs. (1) and (2), kraoe at Se ban In this case Q= x 50, Ty om of Inte kx @ So, () Wen 22 oh gee an ny he cs will move wih ame veo acceleration. (Say w) and clearly m, moves downward. From Newton's second law in projection form (downward for m, and upward for m,) we get : m,8-T,= mw 4 and T,-mg= mw o 54 194 19s 196 197 Th Also Fm © sienoron ano B (19 (9: (m— Noms Mm Mm Giorno ee ‘The force with which the cylinder wall acts on the particle will provide centripetal force necessary for the motion of the particle, and since there is no acceleration acting in the horizontal direction, horizontal component of it remain constant througout ie Using, F, = mw, , for the particle of mass m, = vycosa mikeot?a mes which i the required normal force. Obviously the radius vector describing the poston of the particle relative to the origin of coordinate is Fe attyj= asinoritbeosotj” Differentiating twice with respect the time : We EEL ~ 6? (asin ott bcos wtjS= a a Thus Fe mif= -motr” (@) Wehave ap f Far = [ngie~ mi a , 2008) (©) Using the solution of problem 1.28 (b), the total time of motion, «= - =~ ® Hence using f= cind) Lap] = mgt = 2m (vee (eB is -ve) From the equation of the given time dependence force F= @°#(v-1) at ¢+, the force vanishes, (®) Thus wap f Fa 198 but (b) Again from the equation F= mit? Hx-em or, Ter?) dem md Integrating within the limits for 7%C), Ber- 0?) dew m f av fw-rieaf 2 oP) wee % a(-5)- (5-5) Pw (5-1) eres Hence distance covered during the time interval = x, We have F= Fysin ot or m2 Fy sine or mav> Fysin ordt n integrating, 2% j frm —7 cos ar +, (where C is iteration constant) % When f= 0, v=0, 50 C= =e x tne fi As |cosur <1 $0, v= 52 (1-cos or) 199 1.100 Thus Fot Frsinat Fy . mon maT atte) (igure in the answer sheet), According wo the problem, the force acting onthe puri of mass m is, = Fyoos or oo ao Fi s0, EE. Fomor on de 7 cos ort Integrating, within the limits. = Fe nF av 78 f cos onde ot i 7 sin wr ° i Tes lear ftom equation (1), that after stating at ¢= 0, the particle comes to rest fro the first time at t= z. Fo x From Eq. (1), v= [= am SiN for tx @Q ‘Thus during the time interval ¢= 1/, the sought distance ve i = 22 ff sin wed mw J mor From Eq, (1) Fi Yau” wo | sine | = 1 o> ae @ From te problem F = -7 so mSE= iv ‘Tous mee las dvthy) a On integrating But at $= 0, vav% 50, C= Inv «, ata -Lt oe ve ned 0 ‘Thus for te ye 0 ©) Werwemtbe ry wo ve 87 Integrating within the given limits to obtain v (s): or, ® Thus for (0 Letwe have ” J ’ % so 2 mq) _ ming, [Now, average velocity over this time interval, 1101 mabe kv? on, moka Integrating, withing the limits, ® On integrating So, @ Putting the value of k from (2) in (1), we get A(m=¥) % vovin “2 58 1.402 1.103 From Newton’s second law for the bar in projection from, F,= mw, along x direction we get ‘mg sin a — kng cos a= mw Es vite guina-argeosa, (so a) or, vdv= (gsina-arg cosa) dx o vdv= gf (sina x cosa)de 3 a 1 So, Se g(sinas-* wens) ® 7 a From (1) v= 0 at either eee ‘As the motion of the bar is unidirectional it stops after going through a distance of ey ce una. From (1), £08 Yue 4 (sin ax 7 1 a S008 a) = 0, which yields x= Luna Hence, the maximum velocity will be at the distance, x= tan a/a Putting this value of x in (1) the maximum vel vw Vesinectan a Since, the applied force is proportional to the time and the frictional force also exists, the motion does not start just after applying the force. The body stars its motion when F equals the limiting fricto Let the motion start after time f , then bong, F= any=kmg of, y= So, for f= + fy, the body remains at rest and for £> fy obviously dv me a(t) on, mdvm a(t~h)de Integrating, and noting v = Oat £ = fy, we have for£> fy ieee aioe or ve -4 ° * Thus s= fvdte A fenwrae elt 1.104 While going upward; from Newton’s second law in vertical direction : vdy m2 (mest?) “ay oem ‘At the maximum height h, the speed v = 0, 30 ° H vay Sita fa Integrating and solving, we get, iy @ When the body falls downward, the net force acting on the body in downward direction equals (mg-kv?), Hence net acceleration, in downward direction, according to second law of motion vdv bt vd he Binlie “eB ge 7 v s Thus S-4;-Je« ebm ° ° Ttegrating and puting the value off from (1), we ge, vim vo/ Viekva/mg. 1105 Let us fix x -y co-ordinate system to the given plane, taking x-axis in the direction along ‘which the force vector was oriented at the moment ¢=0, then the fundamental equation of dynamics expressed via the projection on x and y-axes gives, ® @ @) and @ Hence, 1.106 1.07 (b) It is seen from this that the velocity v tums into zero after the time interval Ar, which can be found from the relation, oS m coment, the sought distance, is ar se ff vara Sens fa awe So, <> -SF0(F)a/ ar. ae ° ‘The acceleration of the disc along the plane is determined by the projection of the force of gravity on this plane F,= mg sin a and the friction force fr = komg cos a. In our case ‘ke tana and therefore frm Femgsina Let us find the projection of the acceleration ‘on the derection of the tangent to the trajectory and on the x-axis mv,= Fcos.g~ fr= mgsina (cos @-1) mw, F,~freos = mg sina (1-cos @) tis seen fromthis that w, = — w,, which means that the velocity v and its projection v, differ only by a constant value C which does not change with time, Le. earecl where m= veces The constant C is found from the iaial condion v= My wheace Cov since @ = F lly. Finally we oban ve np/(1 +008). In the cource of time + 0. and v--» v9/2. (Motion then is unaccelerated.) Let us consider an clement of length ds at an angle @ from the vertical diameter, As the speed ofthis element is zero at inital instant of time, its centripetal acceleration i 270, and hence, dN— Ads os q™= 0, where 2 is the linear mass density of the chain Let Tand T+d7 be the tension at the upper and the lower ends of ds, we have from, Remw, (T+ a7) +hdsgsing-T= dds, or, aT +h Rdg gsing= ds w, 1108 61 Iwe sum the above equation for al elements, the term f dT'= 0 because there is no tension atthe fre ends, 0 vm rer f sinodg= dow, f d= At, tee m= (1nd) As w, =a at initial moment 0, wa LW, |= AE (1 — cas 4) So, wal wi|= 8 ( Z In the problem, we require the velocity of the body, realtive to the sphere, which ise moves With an acecleation wg in horizontal direction (say towards lef) Hence itis advsible to solve the problem in the frame of sphere (aon-inertil fame). ‘At an atbitary moment, when the body i at an angle O withthe vertical, we sketch the force diagram for the body and write the second law of motion in projection form F,- mw, my? ox, mgcosO-N-mvpsind= = (1) At the break off point, N'= 0, 0 0, and let v= vaso the Eq, (1) becomes, a Hm F080 nO @ From, F,= mw, vy vedo mg sin 0m 08 0 = m “wm 2A or, vdv= R(gsin 0 +wy.cos0)d0 tag, [vara [neat mentite ga -ouyenaed 5 [Note that the Eq, (3) can also be obtained by the work-energy theorem A= AT (in the frame of sphere) therefore, gh (1 ~ cos 8) + mip R sin y= 3 mg [here mivgR sin 0, is the work done by the pseudoforce (- mi] or, = g (1 ~£08 5) + wo sin Op Solving Eqs. (2) and (3) we get, 24kV 5 + 9k? vy = V2GR7S and 0, ~.cos~1] 2*£VS +k ° 3(1 +k) Hence 6 Lejee 17 1.109 This is not central force problem unless the path is a circle about the said point. Rather here F, (tangential force) vanishes. Thus equation of motion becomes, j= Yom constant 2 mia and, Ee Stora ry We can consider the latter equation as the equilibrium under two forces. When the motion is perturbed, we write r= ra+x and the net force acting on the particle is, Ame x) my A me 2) nye (rytxy Tote eg + *) etm mid ‘This is opposite to the displacement x, if n< 1- |"”® is am outward directed centrifugul force while A is tné inward directed external force). L110 There are two forces on the sleeve, the weight F, and the centrifugal force F,. We resolve ‘both forces into tangential and normal component then the net downward tangential force ‘on the sleeve is, wR sin0(t 72 cos " ( 8 } This vaises for 0 O and for 0 o= eof a a orn A a wR mu*Rsin9 CosO oP R>g. Itur Reg, then 1-2 cose & Mu RSinO=F2 is always positive for small values of @ and hence the net tangential force near 0= 0! ‘opposes any displacement away from it. O= O is then stable. 2 oR Tea? R> g, 1- SF cosdisnegativeforsmall @ near O= 0 and 0 0 is then unstable. However 0 = 0, is stable because the force tends to bring the sleeve near the equi position 0= Oy If R= g, the two positions coincide and becomes a stable equilibrium point. uu qua 113 63 Define the axes as shown with z along the local vertical, x due east and y due north. (We assume We ate in the northern hemisphere). Then the Coriolis force has the components. E, = -2m@xv} = Peo [ v,e0s8 in) 7 v, c088j+ v, cos0F} = 2mor(v, cos0 - v, sind) i” since vj is small when the direction in which the gun is fired is due north. Thus the equation of motion (neglecting centrifugal forces) are = Ime (v, sing - v, cos@), y= 0 and Z Integrating we get j= v (constant), 7= ~g¢ nd Fm 2 vsing f+ 0g ¢? coup ly, xe ov? sing + 4 opt? cose Now v >> gt in the present case. so, s\" 2 xe ovsing(2) = sing = Tem (to the east). ‘The disc exerts three forces which are mutually perpendicular. They are the reaction of the weight, mg, veteally upward, the Coriolis free Dv perpendicular tothe pane of the vertical end long the damete, and mo’? outward long the diamete. Te resultant force is, Fa mV Pre PsQvoy The sleeve is free to slide along the rod AB. Thus only the centrifugal force acts on it. ‘The equation is, ive matr where ve £. Bute yee (2") dr dr\2 5 1a. Lets canta on Pe hea? being the inital velocity when r= 0, The Coriolis force is then, amo Vee oP = 2ma? Viva ‘= 283N on putting the values. oe L114 The dise OBAC is rotating with angular 0! velocity w about the axis OO’ passing through the edge point O. The equation of motion in w routing frame is, mid’ = Fs mo? R's dni? xt» FF, e where Fy, isthe rastan inertia fore seudo force) which is the vector sum of centrifugal 0 A snd Coriolis forces. () AVA,Fy, vanishes. Thus 0= ~2mu? RB + 2m oh a where 7 is the inward drawn unit vector to the centre from the point in question, here A. ‘Thus, vin oR yoy 80, wee Seth (o) ALB Fi= mo? OC + mo BE its magnitude is ma? VaR?— 77, where r= OB. 1.115 The equation of motion in the rotating coordinate system is, mit = P+ mu? Rs 2m (xa) Now,’ i= ROZ+R sino and Wm w’con 027-w' sin 0% iz z 5 = mies | 0 RO Rsinog ?m cos -wsind 0 = Z(OR sin’O ) + oR sin 0 cos 0 5 - wR 0 cos 0 2 Now on the sphere, Fe CRE-Rsn OT + (RO ~Rsin 8 cos 042) + (Rsin Of + 2R cos 0.0 6) &%, “Thus the equation of motion are, m(-R®-R sin? Og?) = N- mg cos 0-+ mu? R sin? 0+ 2mw R sin? 0 ‘(RO ~Rsin Gos 0g?) = mg sin 8 + mu? R sin 8 cos 8 + 2mw R sin 0.cos 0 ‘m(R sin 0g +2 cos 8 0G) = - 20 R 0 cos 0 From the third equation, we get, @= -@ A result that is easy to understant by considering the motion in nor-roating frame. The climinating @ we get, mk & = mg cos 8-N mRO = mgsinO Integrating the last equation, Innis met <8) 116 7 Hence N= 20056) mg Sith body mus fy off for 8» y= cos", exaly a if the sphere were nonotaing. Now, at this point F,y= centrifugal force = mu? R sin 0, = vi ‘ma? R Fag VOR Tae 0 +O RFD «On = VER RR at Sk x 2m = 2 mat RS +E Ve RP ra EE xm 2a? a5 ER (0) When te ain i moving slong 4 meridian ony the Coriolis force bas ltr component nd is magaite (ie the previous problem) Im wveue d= 2mwsinh (Here we have put RO v) 2x, 54000 v5 So, Fa" 2% 2000 % 109 x 2 « $00. = 377KN, (we write Afor the latitude) (©) The resultant ofthe inertial forces acting on the tain i, Fi = -2mo RO cos 02% + (mo? R sin @ cos 6 + 2m wR sin 6 cos 0) 2 + (mo? R sin? 0+ 2m w R sin? 8H) e 1 This vanishes if 0 0, = ‘Thus ve ye, res 1 1 jet -FORsind= -aReosd (We write forte latitude here) ‘Tous the train must move from the east west along te 60% parallel witha speed, 1, 2 19-896: 21, 2 10°41637x 106 158 m/s 417 km ‘We goto the equation give in 1111. Here v= 0 so we can take y= 0, this we get for the motion in the x plane. 1 FoRcosh= = ke wg cos ge? depen bey g(t ee besongite Harms = 2th ease VE ‘There is thus a displacement to the east of 22" (a0 3 2 64x 500% 1 xV 2 = 26m, 13 118 1419 1.120 1.21 Laws of Conservation of Energy, Momentum and Angular Momentum, ‘As Fis constant so the sought work done As Fare F-G3-7) o, A= GI447)-[Qi-37)-G27)] = B44] )-(SF)= 177 Diflerentating » (3) with respect to time wads a ye ely a” Be a” IE (As locomotive is in unidrectional motion) Hence force ating onthe locomotive F=mw = Let, ar v= Oat f= 0 then the distance covered during the first r seconds we 2a ® a @ SVT RE Hence the sought force, F= mw = 2as V1+(S/RY Let F'makes an angle with the horizontal at any instant of time (Fig.). Newton’s second law in projection form along the direction of the force, gives : F = kg cos 0 + mg sin 0 (because there is no acceleration of the body.) ASF tt 47 aitrel wor done by the free F, A a= Fart Fas, (where d= |d71) yy = kong ds (cos 0) + mg ds sin 8 h = ng d+ mg dy. ed : ‘ Hence, A ina aeome f x = kong 1+ mgh= mg (e+ 1). Le 67 1422 Let s be the distance covered by the disc along the incline, from the Eq, of increment of ME. of the disc in the field of gravity : AT'+ AU Ay, 0+ (mgs sin a) = ~ kmg cos cs — mg I kL % 5" Gina keas a ® Hence the sought work Ay= ~ kg [500s +1] 203 ki , Ay = {HME [Using he Eqn.) X—T (On puting the values Ay = -0.05 3 mg 1123 Let x be the compression in the spring when the bar m is about to shift. Therefore at this moment spring force on mis equal to the limiting friction between the bar m, and horizontal floor. Hence x= kg [where isthe spring constant (62y)) Oy For the block m, from work-energy theorem : A= AT = 0 for minimum force. (A here indudes the work done in stetching the spring.) 10, Fr~bu?-kngre 0 o xE= F—kmg @ From (1) and (2), m) Fe tal + 3 11124 From the initia condition of the problem the limiting fiction between the chain lying on the horizontal table equals the weight of the over hanging part of the chan, i Anlg= kX (1m) lg (where 2 is the linear N mass density of the chain) So, eo a) Ta hi 7% Let (at an arbitrary moment of time) the length J ofthe chain onthe table is, So the net friction force between the chain and the table, at this axa moment : AC) 4 S.n kN= bhze. @ ‘The differential work done by the friction forces d= Fo-di™ -f,ds-= -krxg(-ds)= dg Teal (Note that here we have written ds = — dr, because ds is essentially a positive term and 1s the length of the chain decreases with time, de is negative) Hence, the sought work done de ® - Wyden - (1-9) ne - 12 4 furs de= ~(1-n) n Ee -13) ohm 1.125 1.126 1127 The velocity of the body, # seconds after the begining of the motion becomes ‘7% WG ge. The power developed by the gravity (mg) at that moment, is P= mg" v= mg 15+ gt) = mg (gt-vosin a) @ ‘As mig” is a constant force, so the average power eA, mea” te where AF"is the net displacement of the body during time of Hight. As, mgLAF so

= 0 We have wy Ze a, on, v= VaR, 1s defined to start from the begining of motion from rest So, a & = VR Instantaneous power, P= F-o% m (w;i,+ w,4, )+ (VaR t4, ), (where @, and 4, are unit vector along the direction of tangent (velocity) and normal = P= mv, VaR t= mat Hence the sought average power fee fmne Se Hence Pe

AS F, is negative, the force is attractive. (@) We have wu, B= -&y -205 and F, ss, Fe rasit2pyi'nd, Pe WEEE ® Fors cental free, PkF*= 0 Here, TFs QTty Tee 2axT2apy7} = -2pyt-razy@)«0 Hence the fre isnot «cena force. () ASU= as py? ow ww So, Fe We rar and Bo Bho -2By 80, Pa VER - Vide apy According to the problem Fm 2Vabx+ By = C (constant) lus Ls Ls na or, ots pt or, oe @ Therefore the surfaces for which fs coastant an ellipse. For an equipotential surface U is constant, 0, x4 By Cy (constant) a am , ee B= Ke eons Hence the equipotential surface is also an ellipse. Let us calculate the work performed by the forces of each field over the path from a certain point 1 (x, y,) to another certain point 2 (x,y) 4 @ dha Fide ayitdr® ayde or, Ana f yae ee fe (ii) dA= Fodr™ (ait byt )-d7™ axde + bydy 5 n — an feats f ne In the first case, the integral depends on the function of type y (x), ic. om the shape of the path. Consequently, the first field of force is not potential. In the second case, both the integrals do not depend on the shape of the path. They are defined only by the coordinate of the initial and final points of the path, therefore the second field of force is potential. Let s be the sought distance, then from the equation of increment of M.E. AT+ AU = Ay, (0-2mé) roomessina) = = kang cos as « += B/tonastaee) - kw Hence Ay = - kong cosas = Tks tna) Velocity of the body at height 4, vy = V2g(H=F, horizontally from the figure given in the problem), Time taken in falling through the distance r= V% (as initial vertical component of the velocity is zero.) Now s= y,t= Vigil «v2 = VaHh-F) 4 # For Suan (ih) =0, which yes hw Putting this value of hs in the expression obtained for s, we get, Seu" H 116 To comple 2 smoot veil tick ai minimum lg 2 which aie se, mt egal 0. (oe cm proved om ney comer) Th in ot problem body could not reach the upper most point of the vertical track of radius R/2. Let the particle A leave the track at some point O with speed v (Fig.). Now from energy ‘conservation for the body A in the field of, h 1 ma[4—Fa sno | fmt or = gh(l sind) ® From Newton's second la for the particle at the point O; F, = mw, , mi? Nemgsin = ™ ur But, atthe point O the normal reaction N= 0 So, P= sino @ From @) ad (8 snd 3 ad v= VE “After leaving the track at O, the particle A comes in air and further goes up and at maximum height of it’s trajectory in air, it’s velocity (say v’) becomes horizontal (Fig.). Hence, the sought velocity of A at this point. non 2 = vcos (90-6) = vsinO= 5 3 1.137 Let, the point of suspension be shifted with velocity v, in the horizontal direction towards Jeft then inthe rest frame of point of suspension the ball stars wih same velocity horizontally towards right. Let us work in ths, frame. From Newton's second law in projection form towards the point of suspension at the upper most point (say B) Lr « mg+T= Condition required, to complete the vertical circle is that T2 0. But @ J mvj mg 2n+ km} So, yy 4g ® 1138 1139 3 From (1), @) and @) Te M4) ng z0 os, 2 aT Tws vain” Vai Fiom the eqution P= mi, at plat C rem ® t ‘Again from energy conservation Sh = 5 2+ ml © From (4) and () = 35mg Since the tension is always perpendicular to the velocity vector, the work done by the tension force will be zero. Hence, according to the work energy theorem, the kinetic energy or velocity of the disc will remain constant during it's motion. Hence, the sought time t= ©, where s is the total distance traversed by the small disc during it’s motion. % Now, at an arbitary position (Fig) ds (I-R0)d8, ” 50, s=fq-noao B "IRV, It should be clearly understood that the only uncompensated force acting on the disc A in this case is the tension T, of the thread. It is easy to see that there is no point here, relative to which the moment of force T is invarible in the process of motion. Hence conservation of angular momentum is not applicable here. Hence, the required time, ¢ Suppose that AV is the elongation of the rubbler cord. Then from energy conservation, AU, + AU,= O(as AT= 0) =mg(l+ A+ hx Al= 0 or $e Al?—mgAt—mg t= 0 ™ 1.140 144 nes Vine Ae el fr] Al 2 1+V1 "g er ¥ 2x¥ sic te vane V+ BE cry greater ace anv sg vee me(, fz se, a (Vr) ‘When the thread AA is burnt, obviously the speed of the bars will be equal at any instant of time until it breaks off. Let v be the speed of each block and 8 be the angle, which te elongated spring makes with the vertical at the moment, when the bar A breaks off the plane. At this stage the elongation in the spring. Al= [psec 0 ~ fy fy(sec 0-1) @ Since the problem is concerned with postion and there are no forces other than conservative forces, the mechanical energy of the system (both bars + spring) in the field of gravity is conserved, ic. AT+ AU = 0 So, 2(pm']+ pW ec 0? main 0 @ From Newton’ second law in projection form W along vertical direction = 6 img N-+% lg (Sec 8~1) cos 8 Klo Gao) But, at the moment of break off, N= 0. Hence, x ly (see 8-1) 008 = mg Lor Klp-me or, cos = ® «hy Taking c= 52, intact) ads VEE Obviously the elongation in the cord, Al= fy (sec ® ~ 1), at the moment the sliding first starts and at the moment horizontal projection of spring force equals the li So, x, Alsin O= kN 17 m/s. (where x, is the elastic constant). KAL Kg AN From Newton's law in projection form along vertical direction : x, Alcos 0+ = mg. fr N= mg, Alcos 8 bk @ From (1) and (2), pee es 1142 1143 a 17 Aisin O + k Al cos 6 From the equation of the increment of mechanical energy : AU+AT= Ay, Le os (}e.8") =a kong al? % Falteind + boos)” Ae ‘kg Ip (sec 8 Tw Ayn PEED 5695 (on xan et te defeat ine sig eA, when ero AB bas tained he angular voy From Ge seand iw of mation im poecton form Fu mv, ; seAte mage) on, ate MO, Linted al? From the energy equation, Agy= mv? +5 al heat ty+ Aff ed Al = pmo? y+ ADP +5 AP 2 2 Pia 2" leo mo? BUG4T rece ne MO 2-0” ur On solving = Ag ‘We know that acceleration of centre of mass of the system is given by the expression —._ mtomiy Mem ny ee ‘Wy Since We -h Uy op, (m= mit e-em a [Now from Newton's second law F'= miv, for the bodies m, and respectively. r Pome mt @ Fin nte-ne rh fee] tw and Fem mi -mit,— @) | Mog Solving (2) and (3) ce (mim) E aa @ mg, 76 14s 146 Thus from (1), @) and @), ee ee (m, +m) As the closed system consisting two particles im, and of m, i initially at rest the CM. of the system will remain at rest, Further as my = m,/2,the CM. ofthe system divides the Tine joining m, and m at all the moments of time in the ratio 1 : 2 In addition to it the total linear momentum ofthe system at al the times 8 zero. So, py and therefore the velocities of my and m, are also directed in ‘opposite sense. Bearing in mind all these thing, the sought trajectory is as shown in the figure. First of al, it is clear that the chain does not move in the vertical direction during the uniform rotation. This means that the vertical ‘component of the tension T balances gravity. {As for the horizontal component of the tension 7, itis constant in magnitude and permanently directed toward the rotation axis. Itfollows from this that the CM. of the ebain, the point C, travels along horizontal circle of radius p (say). Therefore we have, Tcos0= mg and TsinO= mop Tus p= £429. o8em oF and T= 2K SN eos 6 (@) Lets draw five body diagram and write Newton's second law in terms of projection slong vertical and horizontal direetion respectively. Neos a—mg+ frsina= 0 @ freosa-Nsina= mu? @ From (1) and (2) 88 (francs mg)= mil mg freos.a~ 147 2 So, fra mas a+ Htooea]= os @ () For rolling, without siding, fre kN ‘but, N= mg cos c= mw? I sin oe 1 ins Steoea] (mg cos o~ mo? Tin.) [Using (3)] = Beaman. ve mo? I(cosa+ksina) <(kmg cos amg sina) ‘Thus osVg(k-wna)(itkiana)l = 2 rad/s (@) Total kinetic energy in frame Kis 1 ae-Pp ede PP T= 5m (HV P+ 5m (H-V This is minimum with respect to variation in ¥, when ar’ om 0, be. my (Hf - VP +m, (3-V)=0 we 1 my o mintm Vy Fmt m+ my Hence, it the frame of CM. in which kinetic energy of a system is minimum. (©) Linear momentum of the particle 1 in the K’ or C frame mm Bre m, (~*~) uM See raat mys or, Pie w(0T- i), where, w= aca reduced mass Similarly, Pie w-7) So, [BEl= Bll = B= wv where, yg = 17-7 ® Now the total kinetic energy of the system in the C frame is Bee 2m, * 2m” 2 Hence T= Suede 3H |e a 78 1148 1349 150 To find the relationship between the values of the mechanical energy of a system in the K and C reference frames, lt us begin withthe kinetic energy T of the system. The velocity of the ith particle inthe K frame may be represented as Vf= 97+72. Now we can write 1 Lie cfeasey (tee: T= Fmd Y Fm (H+ 0E)- CHIE) Lets) mit =D pmB TY mY Smee Since in the C frame) m,¥= 0, the previous expression takes the form T- Trims Teimv? (ince according to the problem v= V) a ‘Since the internal potential energy U of a system depends only on its configuration, the magnitude is the same in all refrence frames. Adding U to the left and right hand sides of Eq. (1), we obtain the sought relationship Eedmv? Ex Eshmvi As initially U= O= 0, so, E= 7 From the solution of 1.147 (b) Paras Fe 5 ulat-7I, As way FL ‘Thus T- 2mom te) Velocity of muses m, and my afer & seconds are respectively. Wa Wheat and = wa Hence the final momentum ofthe system, Be mS bmg = mye my im +m) BE = Bet mat, (where, Fe= moh +m, and m= m, +m) And radius vector, Fo Were See? (mite maT)! Aone mem) 2 oirela? age mtn Bee FAP, where Fu MAM 1151 Lise 183 cry After releasing the bar 2 acquires the velocity v,, obtained by the energy, conservation = mda fu? on, noVE @ ‘Thus the sought velocity of CM. demsVn ame mem, —~ (m+) Let us consider both blocks and spring as the physical system. The centre of mass of the system moves with acceleration a = towards right. Let us work in the frame of mm centre of mass. AS this frame is a non-inertal frame (accelerated with respect to the ground) we have to apply a pseudo force m,a towards left on the block m and m, a towards left on the block m, ‘As the center of mass is at rest in this frame, the blocks move in opposite directions and ‘come to instantaneous rest at some instant. The clongation of the spring will be maximum or mq s ‘minimum at this instant. Assume thatthe block <—{m, Logegogy at —?F 1m, is displaced by the distance x, and the block m, through a distance x, from the initial positions. From the energy equation in the frame of CM. aT +U+Ay, (where Az alsd includes the work done by the pseudo forces) Here, AT-0, Unde rn? and mF F(ytn) mtg | mm 1 2, mb ta) F or, Feta = ME so ° 2mF 5 1982 0.06 8 +3 To my Hence the maxi besween the blocks equals: fy + = lence the maximum separation benween the blocks equals 5 + Zayas Obviously the minimum sepation corresponds to zero elongation and is equal to /y (@) The initial compression in the spring AI must be such that after buming of the thread, the upper cube rises to a height that produces a tension in the spring that is atleast equal to the weight of the lower cube. Actually, the spring will first go from its compressed ase state to its natural length and then get clongeted beyond this natural length. Let Ibe the ‘maximum elongation produced under these circumstances. Then xl mg « Now, from energy conservation, pear mg(Al+D+ tal? @ (Because st maximum elongation of the spring, the speed of upper cube becomes zer0) From (1) an Ten, ep slsion oA quale ME (©) Let v the velocity of upper cube at the position (say, at C ) when the lower block breaks off the oor, then from energy conservation. Jame be (al? =I?) = mg (le 2 yk (Al? = 1") mg (+ At) (where = m/e and at = 728) 2 Let, theCM. of the system mv +0 ae At the position C, the velocity of CM; ve (springs two cubes) further rises up to A yoy ‘Now, from energy conservation, Fm b= Omg bre or, = sf. ee 6 Ba" 36" ae" But, uptil position C, the CM. of the system has already elevated by, (Al+ 1) m+0_ 4m bye Sane. ae Hence, the net displacement of the CM, of the system, in upward direction Sm AYc™ AYc + AYea™ Due to ejection of mass from a moving system (which moves due to inert in direction perpendicular toi, the velocity of maving system does not change. The momentum change being adjusted by the forces on the rails. Hence in our problem velocities of buggies change only due to the entrance of the man coming from the other buggy. From the ass 1.156 1187 81 Solving (1) and (2), we get As th and tt we ami og ee MT So, Ht Gy ee GS From momentum conservation, for the system “rear buggy with man” +m) iG= ms Te) +My ® From momentum conservation, for the system (front buggy + man coming from rear buggy) Mi¢+m (i vy) = (M+ m) vp o. Mie oat So, ib tem Mem Oe) Putting the value of i from (1), we get Ge eee Oh me (@ Let if be the velocity of the buggy after both man jump off simultaneously. For the closed system (two men + buggy), from the conservation of linear momentum, Mij+2m(i+¥q)= 0 te =2mit” eared © (i) Let > be the velocity of buggy with man, when one man jump off the buggy. For the closed system (buggy with one man + other man) from the conservation of linear momentum : Om (M+ mo” +m(aeo") @ Let if be the sought velocity of the buggy when the second man jump off the buggy; then from conservation of linear momentum ofthe system (bugey + one man) : (M+ mo = Miz+mUi+ 9) @) Solving equations (2) and (3) we get = _m(2M+3m)ia” > Gem array ° Peet me m Tarem?* Hence v, > vy ‘The descending part of the chain is in free fal, it has speed v= V2gh at the instant, all its points have descended a distance y. The length of the chain which lands on the floor during the differential time interval de following this instant is vd. 1158 1159 Forth incoming chin clement on the Boor: From dp, = F,dt (where y-axis is directed down) 0- Qvvdt) v= F, de on Byw hits -Ddey Hence, the fore exerted on the falling chain cquals Ao? and is directed upward. Therefore ftom third Jaw th force exerted by the falling Fl chain on the table at the same instant of 4-3. time becomes i? and is directed downward. f % Since a length of chan of weight (yg) already lies on the able the total force on the flor is (Zaye) + Gye) = (hye) or the weight ofa length 3y of chain, Velocity of the bal, with which it hits the slab, v= V2-gh y Alter fist impact, v = ev (upward) but according t the problem ¥'= 20 e= 5 (1) and momentum, imparted to the slab, = mv—(-mv')= mv(1 +e) Similarly, velocity of the ball after second impact, Waevaey ‘And momentum imparted = m (v4 v")= m(1+6)ev ‘Again, momentum imparted during third impact, = m(1+e)e7v, and so on, Hence, net momentum, imparted = mv (1 + €) + mve (1 +¢) + mve (1 +e) +... = mv(lte)(itere?+...) mt) =e” (Gom summation of G.P) mV2gh / (n+ 1)/ (9-1) Using Eq, 1) = 02g mvs. (On substitution) (@)_ Since the resistance of water is negligibly small, the resultant of all external forces acting on the system “a man and a raft” is equal to zero. This means that the position of the CM. of the given system does not change in the process of motion. ie 7= constant or, AFZ= 0 ie. Sm, AF 0 o, (Wray + Ai) +M ATE = 0 ee ol” Thus, m(" +I}+MiI= 0, of, T=- (©) As net extemal force on “man-raft” system is equal to zero, therefore the momentum ofthis system does not change, So, O= mT (+O I+ MIRO 11s9 1460 1.161 1162 83 (@)_ Since the resistance of water is negligibly small, the resultant of all external forces acting on the system “a man and a raft” is equal to aero. This means that the posi the CM. of the given system does not change in the process of motion. ie. 7x constant or, AFZ= 0 ie. J mAi= 0 or, Im (Aa + An) + M Ari = emit ea ‘Thus, m(I" +1)+MI= 0, ot, Ie -M (©) As net external force on “man- of this system does not change, 1” system is equal to zero, therefore the momentum So, O= mT O+HZO]+MIB) or, BO= maa @ AST” ( ce Tl) is along horizontal direction, thus the sought force on the raft Mav Mm dv" a me M di Note : we may get the result of part (2), if we integrate Eq. (1) over the time of motion of man or raft. In the refrence frame fixed to the pulley axis the location of GM. ofthe given syiem ce MAT + (M =m) Ming nyt MAF aie ya Bot Af* -A7en a and ATE* A Fagen + Aleem ey rm) en a tows args Ol Note : one may also solve this problem using momentum conservation Velocity of cannon as well as that of shell equals V2gisin@ down the inclined plane taken as the positive x - axis, From the linear impulse momentum theorem in projection form along x-axis for the system (connon + shell) ie. Ap,= F,At: pcosa~MV2gisina = Mgsina At (as mass of the shell is neligible) poosa-MV2glsina Ate Me sina From conservation of momentum, for the system (bullet + body) along the initial direction of bullet mv mM vgs (m+M)¥, oF, v= 84 1.163 1164 ‘When the disc breaks off the body M, its velocity towards right (along x-axis) equals the velocity of the body M, and let the dise’s velocity’in upward direction (along y-axis) at that moment be v, From conservation of momentum, along x-axis for the system (disc + body) meM ® ‘And from energy conservation, for the same system in the field of gravity : mmv= (m+M)v, or v Li? = Lemay v2+L mv? mah Smt = Sma My vie b my? + meh, were hs he Beit of break off pit fom iia! ev S, 1 1 me 1s , Ln Semen ee mvt mah, sing (1) 2 or, vie P= ae 7 oY tem 7 8 ‘Also, if is the height of the dite, from the break-off point, then, vem 2gh" my +m Hence, the total height, raised from the initial level Me 2g (Mm) So, 2g (h +h y= = Wah = (2) When the dise slides and comes to plank, it has a velocity equal to v= V2gh. Due to friction between the disc and the plank the disc slows down and after some time the disc moves in one piece withthe plank with velocity v (say). From the momentum conservation forthe system (disc + plank) along horizontal towards right ive (m4M)¥ or v= SOE Now from the equation of the increment of total mechanical energy of a system : or, (onsen it waned mas) 1165 1166 (®) We look at the problem from a frame in which the hill is moving (together with the disc on it) to the right with speed u. Then in this frame the speed of the dise when it just gels onto the plank is, by the law of addition of velocities, = u + V2gh. Similarly the ‘common speed of the plank and the disc when they move together is ¥ = us VaR Then as above Ay = L(m+My¥? ~ Lv? ~ Lata? a 2m 2 pln [BE wnat A 20h — Sime sont m 20 mh We see that Zia independent of ts and sin fact just - 4 gas in (4). Ths the result obtained does not depend on the choice of reference frame. Do note however that it wll be incorrect to apply “conservation of enegy” formu in the frame in which the hill is moving. The energy carried by the hill is not negligible in this frame. See als6 the next problem. In a frame moving relative to the earth, one has to include the kinetic energy of the earth as well as earth’s acceleration to be able to apply conservation of energy to the problem. Ima reference frame falling to the earth with velocity v,, the stone is initially going up with velocity y, and so is the earth. The final velocity of the stone is 0 = v,—gt and that of the eat is 5 + 2 gt (M i he mas of the ent), fom Newtons thi In, where £ = time of fall. From conservation of energy 2 1nd «Laid + meh = 2, mod + Sand + meh = 3 12 (mo Hence +e (n + ‘i mgh Negectng % in comparison with 1, we get ¥ = 2gh or v, = V2gh ‘he point i this in ears rest fame the effect of carts aclration is of order ™ and ‘can be neglected but in a frame moving with respect to the earth the effect of earth's acceleration must be kept because it is of order one (ce. lange). From conservation of momentum, for the closed system “both colliding particles” moe + my (m, +m)” <. miltmi_ 1GitaPe2aji26H aay x Wh LOT 2S +24j- 60). 74 97° ay mm, 3 ‘Hence |i] = V14+4416 m/s= 46 m/s or, 1.167 1.168 For perfectly inclstc collision, in the CM, fame, final kinetic energy of the colliding system (both spheres) becomes zero. Hence intial kinetic energy ofthe system in CM, frame completely tums into the interaal energy (Q) ofthe formed body. Hence apes? Jn I frame the same result is obtained as seman? aL uit mi 1 2mm ar ns LIEL? + me 133? Lice ges =~ Sulit - a1 (6) Let the inital and inal edocs of mand my are Uf, i and 77 Hf especialy. ‘Then from conservation of momentum along horizontal and vertical directions, we get : im, u;= m,¥, 008 8 ® and my, = my,sin® Q ty Squaring (1) and (2) and then adding them, i myi~ mite) O- Now, from kinetic energy conservation, Lp ate Lm eet mide pmit amy ® 7 aR) m mm ym 4) mom jt) 5 (4) a. (i) -Ee ) So, fettion of kinetic energy lot bythe parle 1, my wt a 2m [Using (4) © mem” em (©) When the collision occurs head on, muy my ma ® and from conservation of kinetic energy, 1169 87 m, (u, ~ v4) ] {Using (5) ™ ©) 4 Fraction of kinetic energy, lost 4 det (@). When the particles fy apart in opposite direction with equal velocities (ay v), then from conservatin of momentum, mur 0m (m,—m)¥ ® and from conservation of kinetic energy, 1 1 ah mite bm Pekin? my u= (m, +m) @ From £4, (1) and (2), 2 mite mm CH ee rd-3mmeo tine Te Lamy 00 (0) Woen they Ay apart smmeicly slave 0 te inal motion ection withthe ue etdivegiacs 2 ear From conservation of momentum, along horizontal and vertical direction, imu, =m; ¥, 608 (0/2) + mv, cos (0/2) @ and ‘my v, sin (0/2) = my vp sin (8/2) on myn mM ® Now, from conservation of kinetic energy, fmide om Sm desma ® From (1) and @), smu, = 008 (8/2) {m, v,+ )- 2m, v, £08 (0/2) 1.170 So, y= 2,008 (0/2) O) From (2), (3), and (4) mami Am, co(0/2) = mie MANGE cos? (0/2) = 142 0° ing oe Ma dene? 2-1 im 403 and putting the value of 8, we get, “b= 2 ing on, 7 If (»4,,¥yy) ate the instantaneous velocity components of the incident ball and (2g ¥zy) ate the velocity components of the struck ball at the same moment, then since there are no external impulsive forces (ie. other than the mutual interaction of the balls) We have usina= vy, vy=0 ‘mu COSCL = mV, +m Vag ‘The impulsive force of mutual interaction satisfies a Ez da Suda Ea For) (Fis along the x axis as the balls are smooth. Thus ¥ component of momentum is not tuansferred.) Since loss of KEE, is stored as deformation energy D, we have wre Do mit~ Smnt~ Bog x 1 or — Lin, 2 = Lav? = boca = Bran? Sowa? =) at mPiPeosta mv? — (mucosa mys,)* ] D zs [ 2atwcosany,— 2m? ] = m(voosa 2) 2 -[(s-~) | We see that D is maximum when ZS 7% and Da.» Bots | Pos 1 cody a 1 a Then no pee Foote § z (On substiuting a= 45" x LY71_ From the conservation of linear momentum of the shell just before and after its fragmentation 1a Wee ey @ where 7f, 7] and fare the velocities of its fragments From the energy conservation anyev?+ ig @ Now Toc Hg = vf 00 = 7-9 @ where 7Z= 7" velocity of the CM. of the fragments the velocity of the shell. Obviously in the CM, frame the linear momentum of a system is equal to zero, 80 attetrno O) ‘Using (@) and (4) in (2), we get Sy PET + P+ OL on 252425; F, cos +293+3 (1m)? =0 © If we have had used ¥5=-¥; - Vj, then Eq. 5 were contain 7; instead of ¥, and so on. ‘The problem being symmetrical we can look for the maximum of any one, Obviously it willbe the same for each. For f,to be real in Eq, (5) 497} 00s°0 = 8(273 + 3 (1 - n) 9) oF 6(n - 1)v" = (4 - c0s"O)7 > Sp 362974203627 FT 80, Bev VSOED oe Fygay=VE=D Hence v2 uu)" 17 ane wv V2 v av(14+V2@-0 =1km/s Thus owing to the symmetry yn)" Yty" Yam (1+VH=T)) = km/s Since, te alien head'on, he ple 1 wil connue moving slong the same Tine ts belre he clin, bt tere wil be a change in he magntde os veloc vetr Tet it stata moving with velocity» and pile 2 with v fer clio, ten fom the contevnon of momentum ce eee eae © an tam te cond, pre on From (1) and (2), 2 vi+u-v)P= O-ne? » shew -2uy+ v= (L=na? o, Dh-2uenute 0 (aa By 0 a vyo Que, =} [eV ]- dua evi) Positive sign gives the velocity of the 2nd particle which lies ahead. The negative sign is comet fot ¥,« So, y= 4u(1-VI=2H) = 5 m/s will continue moving in the same direction. Note that v,= 0. if = 0 as it must. 1.173 Since, no extemal impulsive force is effective on the system “M + m”, its total momentum. along any direction will remain conserved. So from p,= const. mu mum Mv,cos 9 on, v= TG @ and from p,= const My : m= My,sinO of, v= 7-v,sinO= wtanO, [using (1)] Final kinetic energy of the system ‘And initial kinetic energy of the system= 1-7, So, sechange = ETE 190 fated sadt0 te = [wos faa?0-1) x0 and putting the values of O and, we get % of change in kinetic energy ~ 40 % L474 (a) Let the partces m, and m, move with velocities 7] and Ff respectively. On the basis of solution of problem 1.147 @) Be nyg =H [-B| on As wy So, Be nView where w= rH (©) Agsin from 1.147 (6) i. 7-H a So, To FuOG+¥d 1.175 From conservation of momentum Raw +e ° (MY = Han, nfo, + mf? «rs? From conservation of enray Prop pe 2m, ~ Im, * 2m iminaing pj we get A ™ , of mm om met Bt) - arenes enst- 3) ‘This quadratic equation for p,’ has a real solution in terms of p, and cos 0, only if satan 3 ~ mi 3 1.176 From the symmetry of the problem, the velocity of the disc A will be directed either in il direction or opposite to it just after the impact. Let the velocity of the disc A after the collision be ¥ and be directed towards right after the collision. It is also clear from the symmetry of problem that the dises B and C have equal speed (say v") in the directions, shown. From the condition of the problem, né cos Om P= D so, sind= Va=ae /2 @ For the three discs, system, from the conservation of linear momentum in the symmetry direction (towards right) mv= Inv" sinO+ my’ of, v= 2v"sinO+¥ @ 92 417 1.178 From the definition of the cocfticein of restitution, we have for the discs A and B (or C) v'=v'sin 8 ” “ysin 8-0 But e= 1, for perfectly elastic collision, So, vsin@ = v"=vsin8 o 8 Ry ay! From (2) and (3), A ye UL=2 sin? 9) ” (1+2sin? 0) = UOP=2) agin Soar ine CO Hence we have, pil Therefore, the disc A will recoil if v7-+ v3 20, 97, 77,>0 and therefore angle of divergence < 90" Suppose that at time f the rocket has the mass m and the velocity 9°, relative to the reference frame, employed. Now consider the inertial frame moving with the velocity that the rocket has atthe given moment. In this reference frame, the momentum increament that the rocket & ejected gas system acquires during time di, dp’ md wait F dt or, mit= F- yi 79 1.180 1181 According to the question, F’= Oandy = - dm/dt so the equation for this system becomes, As dv 4a" 50, mdv= -udm. Integrating within the limits : ‘Thus, v= win my As dit i” 50 in vector form 9" ~ iin => According to the question, F (external force) = 0 av dm So, nD ae As wit 0, in scalar form, mdv= -udm or, ae a Integrating within the limits for m () i * Hence, m= me As F'= 0, from the equation of dynamics of a body with variable mass; av pdm nem ee a Now di"? 4iand since ZL i we must have |dV"|= vod (because vis constant) where da is the angle by which the spaceship tums in time de So, xn yoda or, dam - od oe 4 1182 1.183 Liss We have a pot, d= ~ pat a Imegating fda fat on, m= my—we ns ° ‘As i> 0's, from the equation of variable mass system (mo- 1) = Fo, Pw ho Fim w) Hence E 0 gw Let the cat be moving ina reference frame to which the hopper is fixed and at any instant time, fet ts mass bem and velocity 7° ‘Then fom the general equation, for vatiable mas system. ae a ‘dt ‘We write the equation, for our system as, Fv a5, Te 7 « 4 oy -F So. a mmnF and = Fon integration. But m= m+ ut Let the length of the chain inside the smooth horizontal tube at an arbitrary instant is x. From the equation, oF mite Feit 1185 as a> 0, Ft w, for the chain inside the tube dawe T where he T Oy Similarly forthe overhanging part =o WA reece ‘Thus mw= F or AAwe hhg-T @ From (1) and (2), Aleehywe hhg o @+Mv oem he dy on, shy ee Goiy Cag eh [As the length of the chain inside the te decreases with time, ds = - de} de or, vdvw gh ae Svan en f 35 on» e Zo h(t) or v= Vier) Force moment relative to point O 5 = ait_ yc N= dt 2b . Let the angle between Mf and N, a= 4S? att = fy 1 MN | @+b%): 2b) then fy ACK VE [MIN] Var +0719 2. . eg eg Veebig ry Vari tg Bee So, 2b f= abt oF, n= VEGs fg cannot be negative) It is also obvious from the figure that the angle ais equal to 45° To when a= big, ie. fo = Va7B and N= 2 at the moment fo, 1.186 1.187 1.188 M- Fp (reer 3a 5 = myst sin(F +4] e 1 spate a(-F): = ymvo gt? cos a (-F'): mvp gt? cos. ~T Thus angular momentum at maximum height Thus M (= 8 iat r= 5 J [(e+3 (@) The dise experiences gravity the force of reaction ofthe horizontal surface, and the force Rof reaction of the wall at the moment of the impact against it. The first two forces ‘counter-balance each other, leaving only the 4 force K. It's moment relative to any point of A the line along which the vector R’acts or along normal tothe walls equal to zero and therefore the angular momentum of the dse relative to any ofthese points does not change in the given process. () During the course of collision with wall the position of disc is same and is equal to FZ, Obviously the increment in linear momentum of the ball Ap’ 2mv cos of Here, AM = 72, x Ap 2 mv cos aft and directed normally emerging from the plane of figure Thus |AM|= 2mv cos a (@) The ball is under the influence of forces 7° and m at all the moments of time, while ‘moving along a horizontal circle. Obviously the vertical component of T'balance m "and He oS ° 3 oe Lass 1.190 °7 ‘0 the net moment of these two about any point becoems zero. The horizontal component of 7, which provides the centripetal acceleration to ball is already directed toward the ‘centre (C) of the horizontal circle, thus its moment about the point C equals zero at all the moments of time. Hence the net moment of the force acting on the ball about point CC equals zero and that’s Why the angular mommetum of the ball is conserved about the horizontal circle. (©) Let abe the angle which the thread forms ‘with the vertical. Now from equation of particle dynamics : Teos a= mg and Tsina= mo? Isina Hence on solving cosa f(a) ‘As |M | is constant in magnitude so from figure. [AM |= 2M cos a where M= |Mil= 1,1 = lem |= mot (on Faw) ‘Thus| AM |= 2mv cos a= 2.mu I? sin a. cos a = tasty ne During the free fall time ¢= « = v2 + the reference point O moves in hoizontal direction (cay towards right) by the distance Vx. In the tansaing frame as 17 (0) =0, 0 Mei 3 = VET AF xm (ge -VF] : = -mVgci'e ma (eb) 5) Ey mv heb) = —mvhE a env, (F Hence [AM|= mva ‘The Coriolis force is.(2m¥" x @). Here Sis along the zaxis (vertical), The moving disc is moving with velocity vy which is constant. The motion i along the xaxis say. Then the Coriolis force is along y-axis and has the magnitude 2m vg on At time the distance of the centre of moving dite from 0 is vg (long x-axis). Thus the torque N due w the coriolis fore is Ne Imvgarvg along the axis. 191 192 “Note : Eq. (1) can be de Bence equating iso BO amit or M= migox? + constant, The constant is irclevant and may be put equal to zero ifthe disc is orginally st in motion from the poiat O. This discussion is approximate, The Coriolis force will cause the disc wo swerve from ttightline motion an ths case deviation fom he sbove ora which wil be substan forage If radial velocity of the particle then the total energy of the particle at any instant is igi ty ME ag Lite Mtoe “ 2 ane w where the second term is the kinetic energy of angular motion about the centre O. Then the extreme values of rare determined by 7 = 0 and solving the resulting quadratic equation 2 pay Me HPP EP + we get From this we see that Enkei) @ where 7, isthe minimum distance from and r, is the maximum distance. Then Joh + 2k} =k (or) Hence, m= y d from the standard expression for kinetic energy and angular ‘momentum in plane poler coordinates : = bmi? hmPo? Taz zmre ‘M = angular momentum = mr”6 ‘The swinging sphere experiences two forces : The gravitational force and the tension of the thread. Now, it is-clear from the condition, given in the problem, that the moment of these forces about the vertical axis, passing through the point of suspension NY = 0. Con- sequently, the angular momentum M, of the sphere relative to the given axis (2) is constant. Thus ‘vy (sin 0) = mv I ® ‘where m is the mass of the sphere and v is it s velocity in the position, when the thread forms an angle © with the vertical. Mechanical energy is sso conserved, a8 the sphere is 1193 194 1195 99 ‘under the influence if only one other force, ic. tension, which does not perform any work, as itis always perpendicular to the velocity. So, J+ mg teas O= Sm? @ From (1) and (2), we get, vos VIRI7E055 Forces, acting on the mass m are shown in the figure. As N= mg the net torque of these two forces about any fixed point must be equal to zero. Tension 7, acting on the mass m is a central force, which is always directed towards the centre O. Hence the moment of force T is also zero about the point O and therefore the angular momentum of the particle rms conserved about 0. Let, the angular velocity of the particle be ©, when the separation between hole and particle m is r, then from the conservation of momentum about the point O, : mar) r= mor) r, or or on 8 Now, from the second law of motion for m, T= F=mo'r Hence the sought tension; On the given system the weight ofthe body m i the only force whose moment is effective about the axis of pulley. Let us take the sense of a of the pulley at an arbitary instant as the positive sense of axis of rotation (z-ais) As M,(0)= 0, 50, AM,= M,()= fN,ae s, Mo fmeRar~ me Let the point of contact of sphere at initial moment (f= 0) be at O. At an arbitrary moment, the forces acting on the sphere are shown in the figure. We have normal reaction N,= mg sin a. and both pass through same line and the force of static friction passes through the point O, thus the moment about point 0 becomes zero, Hence mg sin cis the only force which has effective,torque about point O, and is given by |N|= mgRsina normally emerging from the plane of figure. As Mee 0)= 0, s0, alt= M(y= f Nae Hence, M()= New mg Rsinat 100 1.196 1197 1198 Let poston vectors ofthe particles ofthe system be F and 7” with respect to the poins © and O* respectively. Then we have, Tn +7 a where 73 isthe radius vector of O' with respect to 0. "Now, the angular momentum ofthe sytem relative othe point O canbe writen as follows; M~ SP) - DF 7) + (ex?) [using (1)] or, ue + Gexa), where, p= a Q From (2), ifthe total linear momentum of the system, = 0, then its angular momen- tum does not depend on the choice ofthe point 0. Note that in the C.M. frame, the system of particles, as a whole is at rest. On the basis of solution of problem 1.196, we have concluded that; “in the CM. frame, the angular momentum of system of particles is independent of the choice of the pont, relative to which iti determined” and in accordance with the problem, this is denoted oy. We denote the angular momentum of the system of particles, relative tothe point O, by (M. Since the internal and proper angular momentum ff, in the C.M. frame, does not depend con the choice of the point 0, this point may be taken coincident withthe point O ofthe K-frame, ata given moment of time, Then at that moment, the radius vectors of all the Particles, in both reference frames, ae equal (7 7) and the velocities are related by the equation, ie H+, a where V7 is the velocity of CM. frame, relative to the K-frame. Consequently, we may ie Dm (Peit)= J m(z x) +E mT) on, Mm Bom (72x72), a8.) mite miz, where mS) m, on, Mea 8s (remyt)~ M4 (7x7) From conservation of linear momentum slong the direction of incident ball forte system consists with clldng ball and phere rings mo 9 ® were and v,are the velocities ofall and phere respectively afer cllision, (Remember thatthe collision is head on, AS the callsion is pefecly elastic, from the definition of coca of restintion, Cony ot of, = y= =H @ 1199 101 Soiving (1) and @), we get ye ss directed towards right, Se eae ears) Bie Pi and [Bil= Lal = wlT-7l Also, Fe = -Fie, thus M = 2[7oxPi] c m/z AY »] 2°23 Wg (here is he uit vector in he sense of Fic Pz) Mp Ase A Br, ofa = _myyl Hence iit= In the CM. frame of the system (both the discs + spring), the linear momentum of the discs are related by the relation, p]= ~ p> at all the moments of time, where, Bix Bin Be be ‘And the total kinetic energy of the system, T. $e v2, [See solution of 1.147 (b)] Bearing in mind that at the moment of maximum deformation of the spring, the projection of 7 along the length of the spring becomes 20, Le. Yuu) 0. ‘The conservation of mechanical energy of the considered system in the C.M. frame gives. 2(3}%" 3 <3 (5) 226) ® Now fom the conservation of angular momentum of he system about te CM, 4(8)4)-2(2 boon on, er ee @ Using @) in (1), peal (gf ee 2 5 need je " e 102 1.4 UNIVERSAL GRAVITATION 1.200. We have MP yMm, or r= Um pp er Thus ee 7 ym iF om, (Here m, is the mass of the Sun.) nym, 2x 667 x10" x 7 x10? So T= Pax 66) «1070 197410 vy (49x10) (The answer is incorrectly written in terms of the planetary mass M) = 194% 107 see = 225 days. 1.201 For any planet So, (%) Thus fi) So (By) Ty)? = (127°? = 5-24. 7, at = (MEE) 5 ye omen? . ~ where T= 12 years. m,= mass of ths Sun. Putting the values we get V, = 1297 km/s 8) st] -(#'er = 245 x10 km/s? 103 1.202, Semi-major axis (r + R)/2 Ue sulci the mtn be ang «ec of sem oie "2 fe 7 dosnt depend on een rer)? ae V2 (gain m, is the mass of the Sun) Hen ‘of maximum distance R the time of fall then 1.203 We can think of the body as moving in a very clongated and minimum distance 0 so semi major axis = R/2. Hence if (=)- (2) o ve ra2 T)"(R or = T/ANE = 365 / NE = 645 days. 1.204 T= 22R*7/ Vm, Ie the distances are scaled dovin, R°7 decreases by a factor Zand so does m, . Hence T docs not change. my my mam, 1.205 The double star can be replaced by a single star of mass moving about the centre ‘of mass subjected to the force y mm, / 1°. Then pe 28? 2a? vit Verma so Po. Dam va Pa = (G) an’? = Vy Tan 1.206 (@) The gravitational potential due to my atthe point of locaton of m ye feof pa vm ym my So, Uys mVa~ -*7 ym my Similarly Uge -1 () hows te fcaton of the pnt mass 15 he rn Then te pote AU o an cement of mas dl =r of te rod in te dof the pint mas i Mad ay =~ ym Mae ‘where x is the distance between the element and the point. (Note that the rod and the point mass are on a straight line.) If then a is the distance of the nearer end of the rod from the point mass. ‘The force of interaction is aw Pong mM A(t) i -3) werd ‘Minus sign means attraction. 1.207 As the planet is under central force (gravitational interaction), its angular monientum is, conserved about the Sun (Which is situated at one of the focii of the ellipse) 44 So, mayriemiars om 30 58 ® From the conservation of mechanical energy of the system (Sun + planet), en im 2" Le we or, 38 [Using (1)] Thus, ve Vem AT FAD @ Hence Me myzr= mV2ym, 1G) 1.208 1209 108, From the previous problem, ifr, , r, are the maximum and minimum distances from the ssun to the planet and, , vz are the corresponding velocities, then, say, Ly? Baym? ‘mm, ym, : “OM [Using Ea. (2) of 1.207] tn om er 2a Meine Ea. @) 1 where 2a = major axis =r, +r, The same result can also be obtained directly by writing an equation analogous to Eq (1) of problem 1.191. Eade? Om 2 (Glere M is angular momentum of the planet and m isis mass), For extreme position +0 and we get the quadntic we FP symm no The sum ofthe to roo of this equation are mm, rite ate mm, ta From the conservtion of angular momentum about the Sun. myrslna= my r= Myr of YTL= nh” RANE 0 ‘Thus = constant From conservation of mechanical energy, ymm 1 ymm Lm % 2 7 1 dmg werdsinta_ym, Tae One) Re 2ymr,-veAsina= 0 where = voyo/ 1 my (m, is the mass of the Sun). 106 1.210 12 ‘At the minimum separation with the Sun, the cosmic body's velocity is perpendicular to its position vector relative tothe Sun. If raj, be the sought minimum distance, from con- servation of angular momentum about the Sua (C). vol sip may 0, v= a From conservation of mechanical energy of the system (sun + cosmic body), Lyte MEM Ly? Bf e 2 Lae Bm xg so, We aig 1 z Tain * Br, (using 1) «, VBrdgt Brigg WG = 0 a2 msVameageel —-ym,sVE mee a ae w Hence, taking positive root = (rm nb) [Vie drm F 1] ‘Suppose that the sphere has a radius equal to a. We may imagine that the sphere is made up of concentric thin spherical shells (layers) with radii ranging from 0 to a, and each spherical layer is made up of elementry bands (rings). Let us first calculate potential due to an elementry band of a spherjcal layer at the point of location of the point mass m (say point P) (Fig.). As all the points of the band are located at the distance / from the point P, 80, doe 12M (ee mu of he ed) o om. (seas) (2reninen (ea) -()meo And P= a?4r?-2arcos (3) Differentiating Eq. (3), we get ld» ar sind a0 ® Hence using above equations ao (aar}@ © 1.213 107 Now integrating this Eq. over the whole spherical layer doasoue tat f So doe 1 © Equation (6) demonstrates that the potential produced by a thin uniform spherical layer outside the layer is such as if the whole mass of the layer were concentrated at it’s centre; Hence the poteatial due to the sphere at point P; o fags -tfam~-™ o Tis expen i sna to at Hence he soe poke cag of gavin intron of he patie m and he ‘sphere, = ope =e (b) Using the Eq., Gn UE (wsing Ba. 7) So Gu MF and Fo mGu- ® (The protem as sendy clea iat in he anower sect ofthe problem book). Here we Mpls iteent meio. etm be te mas of the spherical layer, wich imagined vo be made up o rings At Pint idee sphtial layer at dance? fom the ceaue, he greviaional potealal die to fing clement of adus a equ, p= - 2 a (ee By (6) of slain of 1211) nce 2. He G,--Z-0. Hence gravitational field strength as well as feld force becomes zero, inside a thin sphereical layer. (One can imagine that the uniform hemisphere is made up of thin hemispherical layers of radii ranging from 0 to R. Let us consider such a layer (Fig.) Potential at point ©, due 108 1214 (Chi boca alt point ofeach hemp! sell ar equities fom. 2) Hens, = fap= 2 frar= 3 Ry 2R M ence, the wor one by the grvtionl eld foce om he pile of mast m to nemove to ny gen by he frm 7 ‘An mg, tno g = Oa nS Hence the sought work, sumit foes ORY y ‘(The work done by the external agent is - A.) yw) In the solution of problem 1.211, we have obtained @ and G due to a uniform shpere, at a distance r from it’s centre outside it. We have from Eqs. (7) and (8) of 1.211, pe MH and B= ue “ Accordance with the Eq. (1) of the solution of 1.212, potential due to a spherical shell of radius a, at any point, inside it becomes 1M. const. anc 32, = W4. const and G,= -22= 0 ® For a point (say P) which lies inside the uniform solid sphere, the potential g at that point may be represented as a sum. ite” 148 where g, is the potential of a solid sphere having radius r and i the potential of the layer of radii r and R. In accordance with equation (A) x(_M_4,,3)._yM, a (casei )- oR The potential @, produced by the layer (thick shell) is the same at all points inside it.The potential @ is easiest to calculate, forthe point positioned at the layer’s centre. Using Fa. (B) Ga? isthe mass of a thin laser betveen the radii > and r+ dr. Thus faut +042 [EYL 2 where dM = —M an? dro (a )?« © 121s 109 ‘om the -=22 From the Eq. o- 3 c= Me 2 Be Me Gc = Me yap Mi, , (ovtere p= 2, isthe density ofthe sphere) © a ‘The plots ¢ (P) and G (P) fora uniform sphere of radius R are shown in figure of answershect. Alternate : Like Gauss’s theorem of electrostatics, one can derive Gauss’s theorem for erwin inthe orm fZ-aS> -4xy Maas «For caelitin of Za «pint inside the sphere at a distance r from its centre, let us conside radius 7, Then, Gaussian surface of so, ft 7 J x : Inegrting an summing up, we ge 2 = ~My 2 -Heb-2) ‘And from Gauss’s theorem for outside it: G.Anr= -dnyM ot G,= 2M ms foe ‘Treating the cavity as negative mass of density -p in a uniform sphere density +p and aig the superperon prac, te sought Ged engi = Gd, o G favor + -Syacp (where 77 and 7* are the position vectors of an ofbttary point P inside the cavity with respect. to. centre of sphere and cavity respectively) ms = -4er0 110 1216 1217 ‘We partition the solid sphere into thin spherical layers and consider a layer of thickness dr lying at a distance r from the centre of the ball. Each spherical layer presses on the layers within it. The considered layer is attricted to the part of the sphere lying within it (he outer part does not act on the layer). Hence for the considered layer dp4nr?= dF 177°) anrtaroy or, dP4nr?= rs (vere p isthe mean dens of sper) dpe Sayer bes wpa fave 2B yeu (The pressure.must vanish at r = R.) oy pe 3(1- 67/89) yM'/ ARS, Pating p= M/(A/3) aR? Putting r= 0, we have the pressure at sphere's centre, and treating it asthe Earth where mean density is equal to p= 545 «10° kg/m? and R= 64x 10?km wehave, p= 173x10"Pa or 1-72 108 atms, (@ Since the potential at each point of a spherical surface (shell) is constant and is equal to p= ~1%, {as we have in Eq, (1) of solution of problem 1.212} ‘We obtain in accordance with the equation 1 1 U- Sfame~ Sof dm 221m) ge 2k 2R 1 2 (©) In this case the potential inside the sphere depends only on r (see Eq. (C) of the solution of problem 1.214) (The factor + is needed otherwise contribution of different mass elements is counted twice.) 2 = enon . ( in] Here dm is the mass of an elementry spherical layer confined between the radii rand r+dr: dm= (4nr2drp)= ()r* m1 1218 Leto Y © i = circular frequency of the satellite in the outer orbit, A ay aye = circular frequency of the satellite in the inner orbit. So, relative angular velocity = wy « where ~ sign is to be taken when the satellites are ‘moving in the same sense and + sign if they are moving in opposite sense. Hence, time between closest approaches ane “Be lm where 8 sO ef cee tal 2 fe seed cre 45 days (8 = 0) 0-80 hour (6 = 2) YM _ 667 x 10°" x 5:96 x 10 Las OBE Gara ceed Re 2x2 YY 67 tote g oy= oR (# ye Be) 637x108 = 0034 m/s YMg _ 667x10"" x197x 10 wt oe = 5.9109 m/s? OO" RE (4950x10 x 105 ° Toen 0:05 = 1500034: 0.0008 1.220 Let h be the sought height inthe first case. $0 99M 100°" R+Ak ee (+3) ( R R 12, 1.221 1222 1.223 or Ba lied 100 R From the statement ofthe problem, itis obvious that inthis case h- teLmvte bm (y-288 Tie pov de ga Obviously when the satellite moves in opposite sense comared to the rotation of the Earth its velocity relative to the same frame would be wen oe( 22a ‘And kinetic energy @ From (1) and @) @) @ 18 1227 For a satellite in a circular orbit about any massive body, the following relation holds between kinetic, potential & total energy T#-E, U=2E @ ‘Thus since total mechanical energy must decrease due to resistance of the cosmic dust, the kintetic energy will increase and the satellite will ‘fal’, We see then, by work energy theonm dT =-dE =~ dA, So, mvdvaav'vdr or, SH de moe Now from Netow’s law at an abitay radius ffom the moon's cea. or, W-1) ‘where g is moon’s gravity. The averaging implied by Eq, (1) (for noncircular orbits) makes the result approximate. 1.228 From Newton’s second law 2 Te oc ye VE = 167k ® From conservation of mechanical energy Jon «0 or, oo VE 237 km/s? @ 116 In Eq. (1) and (2), M and R are the mass of the moon and its radius. In Eq. (1) if M and R represent the mass of the earth and its radius, then, using appendices, we can easily get vo= 79 km/s and v,= 11-2 km/s. 1.229 In parabolic orbit, E= 0 so Sow? MO 0 08, y= VEE where M= mass of the Moon, R = its radi On the other hand in orbit ype eo yn Wi R (This is just the escape velocity.) Thus av= (1-v2) YE = 0-70 kms. 1.230 From 1.228 forthe Earth surface pe VE and v= VE ‘Thus the sought addtional velocity v= VE (VE-1) = gR(VE-1) This ‘kick’ in velocity must be given along the direction of motion of the satelite in its orbit. Ave y, 1.231 Let r be the sought distance, then " ag Hav tery or Vile (aR =r) on rm Pw 38% 108 bn, 1.232, Between the earth and the moon, the potential energy of the spaceship will have a maximum, at the point where the attractions of the earth and the moon balance each other. This ‘maximum PE. is approximately zero. We can also neglect the contribution of either body to the pE. of the spaceship sufficiently near the other body. Then the minimum energy that must be imparted to the spaceship to cross the maximum of the PE. is clearly (using E to denote the earth) 1233 uz Mam Re hs ney he pci wi cs ovr he ump inte PE nd const ovate to pS owen be mon ad cred on Wht he pobie ses e inen ene) eed for stnig, That eg he we foes ering teat ering ot te spzhip and sce he inte ney of be ges ete om he okt vl aay be pstv te ol ney equi fr standing spear tan bat req forcing Toca i vey We nsime ft eee ewe ly ose tothe moon when ie speci fs al ied stem! vlc on he mom 2M i i VEE wer ye the me ofthe oon ad yao. Tegner Ae» np and since peed of et ass at shan espe erect, and monet weseed whe eed us mst equal he momento afte peep, the copy Ze te gs ected ots thn he inet enemy cep wn Ry ‘Adding the two we get the minimum work done on the ejected gases to bring about the softlanding. pao (te) ‘On substitution we get 13 x 10° id. ‘Assume fist thatthe attraction of the earth can be neglected, Then the minimum velocity, ‘that must be imparted to the body to escape from the Sun’s pul, is, as in 1-230, equal to OF-1)y where v?= yM,/r,r= radius of the earth’s orbit, M, = mass of the Sun. In the actual case near the earth, the pull of the Sun is small and does not change much cover distances, which are several times the radius of the Earth. The velocity v, in question is that which overcomes the earth’s pull with sufficient velocity to escape the Sun’s pull ‘Thus A yy g Me TROT where R= radius of the earth, My = mass of the earth. m(VE~-1) v? Walting v?= 9M /R, we gt vye Vive (WE= 1) = 166 km/s 1s, 1.5 DYNAMICS OF A SOLID BODY 1234 1235 1236 1237 Since, motion of the rod is purely translational, net toryue about the CM. of the rod should be equal to zero. 0 @ Sought moment W= FA F= (ait 5 x(Ai3 B55 = BR's Ab(~K) = (aB~Ab) FE N aB-Ab tnd am of te force t= Ma SBA, Relative to point 0, the net moment of force: We FeeE ere = (alxAjy e(ahe2 = ab i's AB (-B= (ab-AB)E ® Resultant of the external force Fe RR ajar? @ As N-F= 0 (as WF) so the sought arm I of the force F ab-AB Ie N/F= + aoe For coplanar forces, sbout any pont in the tame plane, J) 77x Fy 7% Fog (where Fg» SL F= estat force) o, Megs Fy Tn gh o he am = Mt Here obviously [Fg |= 2P and iis ected toward right along AC. Take the origin at C. Then About C, x. aks GF War) directed normally into the plane of figute. (Here a = side of the square) Thus = FS; dete into the pane ofthe figure. FUE) 6a nage Hence 1, FUP), o6 45 Thus the point of application of force is at the mid point of the side BC. 128 1.239 110 (@) Consider a strip of length dx at a perpendicular distance x from the axis about which ‘we have to find the moment of inertia of the rod. The elemental mass of the rod equals m dna Fede ‘Moment of inertia of this element about the axis lm dmx? = Fdex™ ‘Thus, moment of inertia of the rod, as a whole about the given axis 1 mag me ref tte! (©) Let us imagine the plane of plate as xy plane taking the origin at the intersection point of the sides of the plate (Fig). 0 Obviously dx “Slay Hence from perpendicular axis theorem Mareb? La L+l= 2 (a+b), ‘Which is the sought , moment of inertia. (#) Consider an elementry disc of thickness dr, Moment of inertia of this element about the z-axis, passing through its CM. j= SDR. 9 sae E oc 4 where p= density ofthe material ofthe plate and = art of cos section of the plate, Thus the sought moment of nein o ; ape tye BSE face psp —_o = Epbn*(ass~ xR?) 120 putting all the vallues we get, J, 2-gm-m? 3 (©) Consider an element disc of radius r and thickness dr at a distance x from the point 0. Then r= xtana. and volume of the disc = man? ade R Hence, its mass dm= xx7tna.dep (where | A aR? p= density of the cone = m/52R7h) iA ves ‘Moment of inertia of this element, about the t axis OA, a= dm = (xx? an?ade) = Estat SP stan ‘ade ‘ us the sought moment tia = 2P tanta f x4 Ths ie sought moment oferta = 2 untae 0 i an Hence BE (pig 0 3 1 lamina of an arbitrary shape and indicate by 1,2 and 3, three axes coinciding with x, y and 2 axes and the plane of lamina as x ~y plane. ‘Now, moment of inertia ofa point mass sbout x= axis, dl, = dm y? ‘Thus moment of inertia of the lamina about this axis, =f dy? Similarly; f,= fedm m,) mg-T,= mw= m, BR 9 and T,-m¢= mW a O) Now from the equation of rotational dynamics Bi of solid about stationary axis of rotation. ie. Ti fa W,= TB, forthe eylinder. or, (T,-T,)R= 1B = mR B/2 @ ‘Similua soluti of the above: tic ields 7 12 AncoUs 5¢ mn of the at ‘equations yi jy wf sm, (m+ 4m) 1 my (m 4mm) mg ¥ mag. 324 1.247 1.248 As the systenr(m +m, +m) is under constant forces, the acceleration of body m, an ‘m, is constant, In addition to it the velocities and accelerations of bodies m, and m, a1 ‘equal in magnitude (say v and 1) because the length of the thread is constant. From the equation of increament of mechanical energy i.e. AT +AU= A,,, at time ¢ whe block m, is distance h below from initial position corresponding to t= 0, 1 1( mR?) v? Fem om) 3 (BEE) mghe — moh a (as angular velocity «@ = v/R for no slipping of thread.) But v7 = 2wh So using it in (1), we get 2 (mz ~ kms) 2 (m, +m.) @ Thus the work done by the friction force on m, f Ape imate ima Eo) dam (m,~ kom) 6° m+2(m, +7) In the problem, the rigid body isin translation equlibrium but there is an angular retardation. We first sketch the ffee body diagram of the cylinder, Obviously the friction forces, acting onthe cylinder, are kinetic. From the condition of translational equlibrium forthe cylinder, mgm Ny+kNz; Nz= kN, (using 2), fenc Nyw 8s Ny bE Hence, jo mia Na ko For pure rotation of the cylinder about its rotation axis, N, JB, on -ANR-RN:R= AB, kn A kmgR (1 +k) _ mR? on, = REE on pe Ute (+k )R Ki 7 Now, from the kinematical equation, 07+ 02 +2, A.p we have, o3(1+2)R GE(TsE)g * because w= 0 128 Hence, the sought number of turns, og (1+) R On” Bek( +k) 41.249 It is the moment of friction force which brings the disc to rest. The force of friction is applied to each section of the disc, and since these sections lie at different distances from the axis, the moments of the forces of friction differ from section to section. To find N,, where 2 is the axis of rotation of the disc let us partition the disc into thin rings (Fig.). The force of friction acting on the considered element dfr= k(2nr dro), (where o is the density of the disc) ‘The moment of this force of friction is dN,» ~rdfr= -2nkogr?dr Integrating with respect to r from zero 10 R, we get 2drw -2akoge’. No nastve [ree Eatoes 2 For the rotation ofthe dise about the stationary dr axis z, from the equation N,= 1B, | 2 3, (2R70)R?, 0g dig 4 ‘Thus from the angular kinematical equation @,= 0+ Bt 4 3R oy oe oye (-ME)o ore BE 1.280 According to the question, 12. WO 01, 12 Yo Integrating, WW = - E+ vom oo EE BH ay, Qiong that at f= 0, = @%) Let the flywheel stops at r= fy then from Eq. (1), y= wVvoy e Hence sought average angular velocity 2Voy 126 1251 1.282 aM, Lets se the equation = Nrlative tothe axis through O a For this purpose, let us find the angular momentum of the system M, about the given rotation axis and the corresponding torque N,. The angular momentum is sa oom (Ben) tto Ly is [where f= 5°R? and v= wR (ao cord slipping)} aM, | (MR? e A (ME na?) a ‘The downward pull of gravity on the overhanging partis the only external force, ‘which exerts a torque about the z-axis, passing through O and is given by, nF Hence from the equation Thus, Note : We may solve ths problem using conservation of mechanical energy of the system (ovlinder + thread) in the uniform field of gravity. (@) Let us indicate the forces acting on the sphere and their points of application. Choose positive direction of xand @ (rotation angle) along the incline in downward direction and in the sense of @ (for undirectional rotation) respectively. Now from equations of dynamics of rigid body ie. F,= mw, and N,,» 1,8, we get: img sina f,= mow ® snd frre Emn?p ® But rs kg cosa. N fi @ I adatom, he ssence of aipying proves gy a Afr the kinematical realtionship between the seston w= BR @ D The simultaneous solution of all the four Mn uations yields “ct 2 2 keosaz Tsing, or kz Fauna (©) Solving Eqs. (1) and (2) [of part (a), we get : 1253 1.254 127 wen Sesina. nn 58 As the sphere starts at r= 0 along positive x axis, for pure rolling. nem mete Sesinat 6 Hence the sought kinetic energy bth 2 mR? ot Lm? (as wo v, T= dts omn?a?= Zo? om ¥/R) at? 7 A(t ys - om (esa) = ines (@) Let us indicate the forces and their points of application for the cylinder. Choosing the positive direction for x and @ as shown in the figure, we write the equation of motion of the cylinder axis and the equation of moments in the CM. frame relative to that axis ie. from equation F, =, mw, and N,= J, By me =F me 2m As there is no slipping of thread on the cylinder w.= BR From these three equations = Ee = 280.5510? ad/s? To Ta 13N, B= 28 5x10? rnd/s ©) wenne po 2 Pa Fe F-) 2 (2z)-2 : ne Ga) amet Let ws depict the forces and their points of application coresponding tothe cylinder attached With the elevator. Newton's second law for solid in vector form in the fame of elevator, ives: 27+ mag's m(-) = mie @ ‘The equation of moment in the CM. frame relative to the cylinder axis i.e. from Ne= [Be 2 mR? wi PTR» BOR [as thread does not slip on the cylinder, w’= BA ] tte 128 on, rem AST tw s0 in vector form 7. ® Solving Ex (1) and (2), = on Fo 2 Lm@ei. 1.255 Let us depict the forces and their points of application for the spool. Choosing the positive direction for x and @ as shown in the fig., we apply F,~ mw., and N,:~ 1,8, and get Oo =) and sought force mg sin a= T= mw; Tr= IB “Notice that ifa point of solid in plane motion is connected with a thread, the projection of velocity vector of the solid’s point of contact along the length ofthe thread equals the velocity of the other end of the thread (if it is not sacked)” Thus in our problem, v,= vp but vj 0, hence point P is the instantaneous centre of rotation of zero velocity forthe spool. Therefore ¥,= Gr and subsequently w, = Br. sy Solving the equnons sauna, we gt = 88 omit ‘, we 9. 1.256 Let us sketch the force diagram for sold ylnder and apply Newons second la in projection frm along andy axes Fig): Jr, + frz= mw, a) and N,+N,-mg-F=0 or Ny, +N,= mg+F @ Now choosing positive direction of @ as shown in the figure and using Na. = 1B we get FR~(fr,+ fro [2s for pure rolling w= BR J. In addition to, frst frzsk(N +N) o 1287 1.258 120 Solving the Eqs., we get re dkmg gp 3kme 2-3" % Pom" 223k K(M, +N) and Wemax) zk ET png 4 248]. 2k Soot Sfaeo a) 2 (@) Let us choose the positive direction of the rotation angle @, such that w., and fi have identical signs (Fig). Equation of motion, F,= mw, and N,.= 1. , gives : F cos a fr= mig: frR~Fr= I,,= ymR?B, In the absence of the slipping of the spool w= B,R Pom heat m= EB, wie > (©) As static friction (fr) does not work on the spool, from the equation of the increment ‘of mechanical energy A,,= AT. “lel? Bet Ki F Aaam 3m + ZymR? pmatye AE. = pm ty 2wx= gmarn2m (me) Fea - 2 fr 2m(+y) mg, Novetas a eos c= r/R, there is no roling and for cos 4<7/R We <0, the spool will move towards negative sans and rtte in amiclclvise sence. For the cylinder fom the equation N,» 18, about i stationary ais of rotation, mw a ane p ape wy Forthe rotation ofthe lower ylnder fom the quan N= 1B mg og pratt a= EB on p= Tap Now forthe transatonal motion of lower ‘pliner om the Eq. Fe = mise mg-2T= mw, @ As there is no slipping of threads on the cylinders wom Bir+ firm 2 Br ® 130 1.259 1.260 ‘Simultaneous solution of (1), (2) and (3) yields = 7B, 7. lat us depict the forces acting on the pulley and weight A, and indicate positive direction for x and g as shown in the figure. For the cylinder from the equation F,= m y and N= 1eBy we get Mg+T,-2T= Mw, o and 2R+T,2R)= 1p = @ ) For the weight A from the equation F,= mm, mg— T= ming ® As there is no slipping of the threads on the pulleys. Wy We +2BR= w,+2W.=3w, (4) ‘Simultaneous solutions of above four equations wives : (+3mg (+ om+ 7) (8) For the translational motion of the system (m,+ m,), from the equation : F,= mig, Fea (mtmy we 0% Wem F/m, +1) ® [Now for the rotational motion of cylinder from the equation : Noy my? oF ee a) But ws we Br, So 2F F(3m+2m) a 28 FGms2m) Gy em my” momem) ©) (b) From the equation of increment of mechanical energy : AT= Ay Here AT= TQ, 80, TO Ace ‘As force F is constant and is directed slong x-axis the sought work done. a= Fe (where x is the displacement ofthe point of application of the force F during time interval r) 1.262 131 2 + -A(3 2). 2H Om+2m) _ oy 2K" |" Damm, +m) (osing Ea. @) Alternate : T(t) * Tyanauon (+ Trtaan 1 Ft - rim.) (Gem) * ant PA Qm+2m) oP 2 my (my, ma) ‘Choosing the positive direction for x and @ as shown in Fig, let us we write the equ of motion for the sphere F,= mw,, and N,.= 1, B, frm mig; frr= 2m? p (isthe acceleration ofthe CM. of sphere.) For the plank from the Eq. F,= mw, F-fa mm In addition, the condition for the absence of slipping of the sphere yields the kinematical relation between the accelerations : wy W)+ Br ‘Simultaneous solution of the four equations yields : and w,= 30, (@) Let us depict the forces acting on the cylinder and their point of applications for the cylinder and indicate positive direction of x and @ as shown in the figure. From the ‘equations for the plane motion of a solid F,= mw, and Nz.= 1,8, kg = mie oF Wee bg a mR? oe ie wlng t= Apr or pw -24f ® Lette cylinder stars pure rollingatr= fy after releasing on the horizontal floor at t= 0. From the angular kinematical equation £ ra Gee or we ay -28 mn 2 RE © a From the equation of the linear kinematics, i "aT 7 or v= 0+ KE ty @ 132 1263 1.264 But at the moment £= fy, when pure rolling starts v,= aR s0, tear (00-2 Bo) oR Thus ao Sie (©) As the cylinder pick, up speed til it starts rolling, the point of contact has purely transatory movement equal tow, in the forward directions but there i also a backward movement of he pit af conn of apne (opty ~ 4 )R. Bere of sing the net displacement is backwards. The total work done is then 1 1 Ay = kg [rea - (ovo 5 Baa] ‘The same result can also be obtained by the work-energy theorem, Ay = AT. Let us write the equation of motion forthe centre of the sphere atthe moment of breaking-of: mR +1) = mg cos 8, ‘here vis the velocity of the centre ofthe sphere at that moment, and 0 isthe corresponding angle (Fig.). The velocity v can be found from the energy conservation law : 1 1 gh = > mi? + > 10, where is the momeat of ina ofthe apere I felative tothe axis passing through the spere’s af cx te» 2 nan, v= or; he (R +r) (1 -cos 8). From te at egutin we bi on Vi0gReniTr. 0} Since the cylinder moves without sliding, the centre of the cylinder rotates about the point ©, while passing through the common edge of the planes. In other words, the point O becomes the foot of the instantaneous axis of rotation of the cylinder. Itat any instant during this motion the velocity of the CM. is v, when the angle (shown in the figure) is B, we have 1268 were Ais the normal reaction of the edge N or, vm gRooep - MR 1 Wie Romp - ” From the enexgy conservation law, 1,%- mg R (1 ~cos 6) (ey) On But fe mat = Ente, 7. (Grom the parallel axis theorem) LP» Tins, w= Br ben -cos) ® From (1) and (2) v= EE (reap -4) - ME The angle inthis equation is clearly smaller than or equa to cso puing B= a we get = BR cosa—4) - MR v= Sa - where No is the corresponding reaction, Note that N’® No. No jumping occurs during this tuming ifN > 0. Hence, vp must be less than Vag WES a toa = VES (1 cos a 4) Clearly the tendency of bouncing of the hoop will be maximum when the small body A, Will be a the highest point of the hoop during is rolling motion. Let the velocity of CM. of the hoop equal v at this position. The static friction does no work on the hoop, s0 from conservation of mechanical energy; E, = Ey : 2 ade bn g2( 1 Lge dng2(® o ovdmgedmae(3) =mgR= Sma? hme hme (§) +mgR = a= f-26R ® From the equation F, = mw, for body A at final position 2 : mg+N'= mo?R= mi) @ —, 134 1.266 1.267 1.268 As the hoop has no acceleration in vertical direction, so for the hoop, N+N'= mg @ From Eqs. (2) and (3), 6 [As the hoop does not bounce, N= 0 6 So trom Es (0, (8) and @), Sh—v ‘ SeR eS oe 8gRa Hence vs OER ‘Since the lower part of the belt is in contact with the rigid oor, velocity of this part becomes zero. The crawler moves with velocity v, hence the velocity of upper part of the belt becomes 2v by the rolling condition and kinetic energy of upper part 1 Bl (2v)’ = mv’, which is also the sought kinetic energy, assuming that the length of the belt is much larger than the radius of the wheels. ‘The sphere has two types of motion, one is the rotation about its own axis and the other is motion in a citcle of radius R. Hence the sought kinetic energy 1 t-} where /, is the moment of inertia about its own axis, and J, is the moment of inertia about the vertical axis, passing through O, eal p at oth ned ® But, t= Em? and J,= Z m?+ mR? (using parallel axis theorem) @ In addition to y= Zand = @ Using (2) and (8) in (we gt 7? = Em (+23) For a point mass of mass dm, looked at trom C rotating frame, the equation 1s amit’ = T+ dm oP 7” +2dm 9" xi) where 7” = radius vector in the rotating frame with respect to rotation axis and v”' = velocity in the same frame. The total centrifugal force is clearly F y= nr” = mR, BCs the radius vector ofthe CM. of the body with respect to rotation axis, also Ean amit x where we have used the definitions mi= J dm7” and miy = dno” 126 1.270 Consider a small clement of length dx at a distance x from the point C, which is rotating in a circle of radius r= xsin 0 Now, mass of the element = (ile So, cenit fre ating on semen ~ (Bacar nd momen o is ae ane, lav|= (FJarotzsne zea sin 20x° de and hence, total moment 2 Let us consider the system in a frame rolating with the rod. In this frame, the rod is at rest and experiences not only the gravitational force m gand the reaction force R, but also the centrifugal force Fey. Inthe considered frame, from the condition of equilibrium i.e. No, = 0 or, Neg me 5sin0 where Nay is the moment of centrifugal force about O. To calculate Nig , let us consider an clement of length de, situated at a from the point O. This element is subjected to + bvizonl pseudo fre (2) dr xan The moment of this pseudo force about the axis of rotation through the point O is aig= (2) aeaesndscaso 7 no = Me sinc Pdr mot? og femora ri sin cos 0 @ 1 follows fom Eqs (1) and @) that, : af wots (2) a oer 26 ° 136 1am 127 1273 When the cube is given an inital velocity on the table in some direction (as shown) it acquires an angular momentum about an axis on the table perpendicular tothe inital velocity and (say) just below the C.G.. This angular ‘momentum will disappear when the cube stops and this can only by due ta torque. Frictional forces cannot do this by themselves because they actin the plain containing the axis. But if the force of normal reaction act ecoentrcally (@s shown), their torque can bring about the Vanishing ofthe angular momentum, We can calculate the distance Ax between the point of application ofthe normal reaction andthe C.G. ofthe cube a follows. Take the moment sbout Initial velocity — Axis to the C.G.ofallte forces, This mastvansh because Initial a the cabe doesnot tim or tumble onthe table. angular inital velocity ‘Then if the force of friction is fr ‘momentum fr 3 N Ax But N= mg and fr= kg, so Axe ka/2 In the process of niotion of the given system the kinetic energy and the angular momentum relative to rotation axis do not vary. Hence, it follows that IM? oad 2p yr), LM? Ze oom pmol sv 47 (© isthe final angular velocity of the rod) Mie Me 2 and ee Co rmo Fm the cq we din : on /(t Ba v= ogt/ Vir 3mm Dye to hitting ofthe ball, the angular impulse received by he rod about the C.M, is equal top wis the angular veloc aqtied by he rod, we hve Mega D ae 2°" 2 mi ® In the frame of CM, the rod is rotating about an axis passing through its mid point with the angular velocity @o, Hence the force exerted by one haf on the other = mass of one half x acceleration of CM. of that par, in the frame of CM. mrt) Ot, wy [eile ate Bean 137 1274 (a) In the process of motion of the given system the kinetic energy and the angular 1275 momentum relative to rotation axis do not vary. Hence it follows that Line Ly? 4 (MI?) 2 Ste bed) Lo gy t ME. and rw b= my oe From these equations we obtain ae °" Ta +4 mM 3m- 4M) 3m aM (b) Obviously the sought force provides the centripetal acceleration to the C.M. of the tod and is and As 0” tt itso im vector form 7” = Fyn rity = My?! Sav? 2" eam ye (2) About the axis of rotation of the rod, the angular momentum of the system is conserved. ‘Thus if the velocity of the flying bullet is v. rte [n? ME) on Om asm cM ® (i Now from the conservation of mechanical energy of-the system (rod with bullet) in the 3m field of gravity 3(meo) P= (Mem) gh cosa.) @ [ecase CO, od ies bythe beigt £1 cna) } Soving 1) nd), gt v= EVE mg ate VE. (b) Sought Ap = [meanea(oz)]-m we of he vey ofthe ule and oi eq evel of CM. of he od aftr the impact, Putting the value of v and o we get dmv I sin & spe Lmve MVE sin ‘This is caused by the reaction at the hinge on the upper end. 138 (© Let the rod starts swinging with angular velocity «’, in this case. Then, like part (a) ‘This vanishes for 1.276 (a) As force F on the body is radial so its angular momentum about the axis becomes zero and the angular momentum of the system about the given axis is conserved. Thus Sotertonten (eae Olt ae (&) From the equation of the increment of the mechanical energy of the system : AT#A,, 2 ME, cmayk?= ME AMR? 22 og Aa Putting the value of @ from part (a) and solving we get mogR?/ om ae BEE (28) 1.277 (8) Let z be the rotation axis of disc and @ be its rotation angle in accordance with right-hand screw rule (Fig.). (@ and ¢ are to be measured in the same sense algebraically.) AS M, of the system (disc + man) is conserved and M, siz) 0, We have at any instant, = E42 om (S)ao( 42) a] 8 « ‘On integrating, fave Sata dy , er ¢ o tm me This gives the total angle of rotation of the disc. 139 (©) From Ea. (1) mde (_m_\vuy ting with respect to time z @e m_)idvcr 2 R & mymR dv (t) Hence Rome 1.278 (a) Frome the law of conservation of angular momentum of the system relative to vertical axis 2, it follows that: Aoythoy= (h+h)o, Hence 0,* (10, +1, 0) /(1 +h) a [Not that for «,> 0, the corresponding vector @ coincides withthe poitive direction to the 2 axis, and vice versa, As both dises rotates about the same vertical axis 2, thus in vector form. BB+ /(h+h) However, the problem makes sense only if, tt, or @, ta (©) From the equation of increment of mechanical energy of a system: Ay, = AT. 1 1, 242 = Lu+hyo2-bhog+dnod Using Eq. (1) hk : Me™ TT Izy (ee Wa) 4 1.279 For the closed system (disc + rod), the angular momentum is conserved about any axis. ‘Thus from the conservation of angular momentum of the system about the rotation axis of rod passing through its CM. gives : Le why me mar mat ® 140 1.280 (is the final velocity of the dise and « angular velocity of the rod) For the closed system linear momentum is also conserved, Hence my = my! + mv, @ (where v, is the velocity of CM. of the rod) From Eqs (1) ~e we get 10 na vee Applying conservation of kinetic energy, as the collision is elastic Ly? Lamy? yyy 2-4 2 ME 2 gma gy a gamitg Tre @) or ove dnd and hence v4’ = 4y, ih Then 4 4- 12v ve Dy and o# aan Geni { IC Vectorially, noting that we have taken #™ parallel to 9” So, "= 0 for n= 4 and ZY tv for n>4 Orton ‘See the: diagram in the book (Fig. 1.72) (@) When the shaft BB" is tuned through 90° the platform must start turing with angular velocity © so that the angular momentum remains constant. Here Jo% (+19) 2 fyay of, = PE “The work performed by the motor is therefore 22 1 1Re} llth) Pa SP If the shaft is tumed through 180°, angular velocity of the sphere changes sign. Thus from ‘conservation of angular momentum, TQ=Iyay= Iyoy (Here ~ fy is the complete angular momentum of the sphere i.e. we assume that the angular velocity ofthe sphere is just ~o). Then 0 a= 4% and the work done must be, ligt Lrat+tia}-Lhaj= 1281 1.282 141 €) Ine ase, fa po, the angle neni ve of he pe spss wha apr veeiy ie re tug fom BE nate ‘The total centrifugal force can be calculated by, 4 my? 1 2 SBorea- dbs ‘Then for equilibrium, 0 i 4 R (1-1) 5= m8 aut, 1 7,= mho? Ths 7, vais, when vie, 7 2, ge VE = onus i ° fon VE menu ‘ 0 Then Ty= mg-P = 25N ‘See the diagram in the book (Fig. 1.71). (a) The angular velocity & about OO can be resolved into a component parallel to the rod and a component « sin® perpendicular to the rod through C. The component parallel to the rod does not contribute so the angular momentum 1 Pasi Mx Losind= mo sind Also, M,= MsinO= T>m1? osin?O This can be obtained directly aso, (0) The modus of 8 doesnot change but the modulus of the change of Mis | AM |. | A= 2mn(90-0)= dpm isco (©) Here M, = Moos0 = Fo sin0 cos8 wdt 1252 ink = Feosind cos0 SB = 5 mi? w? sin? Now dt as M precesses with angular velocity w. 142 1.283 Here M= Io is along the symmetry axis. It has two components, the part [« cos@ is constant and the part M, = I sin® presesses, then | = Fosind o' = mglsind cate prcation eguency= lo ne (0) This force isthe centripetal force due to precession, Itacis inward and has the magnitude Ll [Smo] = me’*tsao = 208, fis the distance ofthe ish clement fom the axis. This isthe force that the table wil exert on the top. See the diagram in the answer sheet ML ML \ ™} 1284 Sete diagram inte bok Fe 1.7 The monet of ners of eds sow fs symmenty xs mA I e ang veloc ofthe cis then he agus momentum tm Ao, The pcesion geny being 227, Gi) 1 Rox we tive |Z ]- fenton This must equal m(g+w)/, the effective gravitational torques (g being replaced by ‘g + w in the elevator). Thus, on 24")! «500 nas Rn 1.285 1.286 1.287 1.288 1.289 14a ‘The effective gis Vg"+ w™ inclined at angle tan” * 4F withthe vical Then wit eference 10 the new * vertical” we proceed as in problem 1-283. Thus on BE 08 nar The vector forms an angle @ = tan! = wie scm verte Te moment finer the spheres 2 and enc the vale of ng momento is Ze, Sc pecese a sped te wxgu regu! 2 Roo! 2nttoa!« PI Lape’ so, P= Bata! = 5008 (Te oe Fmt be vee The moment of na is Lar? and angular momento sre. The ale ects about a horizontal axis making an instantaneous angle, on ean This means ates vr posession wi nt of peceson “The maxima value of this is 7". When the angle between the axle and the axis is at its maximum T value, a torque Io @ ‘gyroscopic force will be ‘The revolutions per minute of the flywheel being n, the angular momentum of the Oywhee! is 1x 2a. The rate of precession is = ‘Thus N = 22INV/R = $97 KN. em. ‘sinh rvs pb cove mast came in py. This canbe done i toe, 22 eon he sin post dct in don whe ela and er forces. The force on the outer rail is increased and that on the inner rail decreased. ‘The additional force in ths case has the magnitude 1-4 KN. m. 144 1.6 1.290 1.291 1.292 ELASTIC DEFORMATIONS OF A SOLID BODY ‘Variation of length with temperature is given by I= (1 +adt) or Le adtme @ g, Thus = GALE, which isthe sought stress of pressure. Putting the value of and trom Appendix and taking Ar= 100°C, we get o= 22x10 aim, But e (@) Consider a transverse section ofthe tube and concentrate on an clement which subtends an angle Aq at the centre. The forces acting on a portion of length Al on the clement are (1) tensile forces side ways of magnitude GArAl The resultant ofthese is 2odrain Sm etrlag diy ovat te ene () The fede id rome = pra Sine test tle, We Bt Pau” Og wer te matinam tse ft Putting the values We get Pyay = 19-7 atmos. (©) Consider an element of area dS = x (7 A@/2)* about z-axis chosen arbitrarily. There are tangential tensile forces all around the ring of the cap. Their resultant is er pas( 2) - oas( $2) ran 2 Ao z Let us consider an clement of rod at a distance x from its rotation axis (Fig.). From ‘Newton's second law in projection form directed towards the rotation axis == (dm) o7x= Fa?x de On integrating mo 7 +C (constant ) 148 But at Thus Hence Thus Tyg = 2 (at mid point) me Condition required for the problem is Trax ™ SOn sop 0e 3VEE ° Hence the sought number of r ps a 2 1v2 using the table n= 08 x 1071ps } 1.293 Let us consider an element of the ring (Fig:). From Newton’s law F, clement, we get, rise (Sao)er (een 10198 ‘mv, for this So, T- 20% Condition for the problem is T go, on ms, rt Om OY Ont a "Sa (arp) or “Thus sought number of rps i or, Using the table of appendices m= 23ps 11294 Let te point O desend by the distance x (Fig). From the condition of equilibrium of point °. 2rsind= mg or T= arg Bvunyex @ Tr a won Bor Te aE Now, (G72 en 4 G) (Co here is stress and ¢ is strain.) 1468 In addition to it Vanes -$ 2 e is vie 7) From Eqs. (1), (2) and (3) eet * Ed? asxe t “pat Se rf tefl ra = F022 Conere p isthe density ofthe copper) Volume of a solid cylinder =arl 2rarl ora 2ar, Al ve + ® VO UaPD Soar or But longitudinal stain A//l and accompanying lateral strain Ar/r are related as So, Arey At ® Using @) in (1), we get av _ al o-fa-w ® Al -F/aP But a (Because the increment in the length of eylinder A/ is negative) a =F Gy, ae Vi apE OP 148 1298 1.299 =I Thus, ave = a-20) Negative sign means that the volume of the cylinder has decreased. (@) As free end has zero tension, thus the tension in the rod'a its lower end vestical distance y from T-Tay 0 Let al be the elongation of the element of length dj, then a Pay - ome paydy/E (where p is the density of the copper) ‘Thus the sought elongation 1 als fat psf 2. dogie ® (tte og ent) sain =, he campy el (mpi strain is given by often ne @ ‘Then since V = x/71 we have AV 2ar Al vor tT al = 4-29) [Using @) where 4 is given in part (8), w i the Poisson rato for copper. Consider a cube of unit length before pressure is applied. The pressure acts on each face. ‘The pressures on the opposite faces constitute a tensile stress producing longitudianl com- pression and ner extension. The compressions i Zand the lta extension sw ‘The net result is a compression E (1 ~2y) in each side av Hence $F = - 2 1-24) because Av at 221-21) because fom symmetry AY = 3 4! 1300 149 Let us consider a cube under an equal compressive stress ©, acting on all is faces. a P iB AV Then, volume stain = - 44. $, ® were isthe bulk modulus of elasticity o_30 So ‘ia | a-2y) , B= 3k0-2n)= 30-2 (es ke F ; 8 8 we} 8 and pare bot to seman pve ‘A beam clamped at one end and supporting an applied load at the free end is called a cantilever. The theory of cantilevers is discussed in advanced text book on mechanics. The key result is that elastic forces in the beam generate a couple, whose moment, called the moment of resistances, balances the extemal bending moment due to weight of the beam, load ete. The moment of resistance also called interal bending moment (LBM) is given vy LBM. = EUR Here R is the radius of curvature of the beam atthe representative point (x, ).J is called the geometrical moment of inertia i-f2e othe cot secon relative ote at psing ough tenet yer wich remains unstretched. (Fig.1.). The section of the beam beyond P exerts the bending moment N(@) and we have, Ne) If there is no load other than that due to the weight of the beam, then N@)= dpg-2"bh rxg e7 where p= density of steel Hence, at x= 0 (i; Ser" (R),” 2ET Here b= width of the beam perpendicular to paper. 150 1.301 We use the equation given above and use the result that when y is small 1idy, @y_ NG) RT ae ™ ET (@) Here N (x)= Ny is a constant. Then integration gives, where we have used y= 0 for x= 0 to set the constant of integration at zero, This is the ‘equation ofa parabola. The sag ofthe free end is Nl? de yon = TS (©) In this case 4 (@) = F(=1) because the load F at the extremity i balanced by & similar force at F directed upward and they constitute a couple. Then dy F-») ae El * dy_ F(le-2/2) Tegrating, Fe 0, As before C, = 0. Inegrating again, using y= 0 for x= 0 Here fora square cross section In f Paden at 1.302, One can think of it as analogous tothe previous cease but with a beam of length 1/2 loaded upward by a force F/2. Fe Thus a ‘On using the last result of the previous problem. 1.303 (a) In this ease N(x) = $ pg 6 (t—x)* where b= width of the gider Also I= b/2 Then, 1.306 151 Integrating, wang 0 tr = 0. Ain negaing eee y- SoR(EE m0 tee = SPs. I 30 3 a i” ar 2 (®) As before, ag N (2) where V(x) is the bending moment due to section PB. ‘This bending moment is clearly Wo f wage -wi ara) + ofut-ate#) wrens of) 2 Gere w= pg bk is weight of the beam per unit length) new meri, 21 2-24). gee 2 ——> aN Tn Be otrsehege wi's wy «enw series, Ety= fe -24 or since 26 nade As y= 0 for x= 0, ¢,~ 0. From this we find Set fay. Soa = yee ne SY" ere ye-) [a1 oe “The deflection of the pate can be noticed by going to a co- rotting frame, In this fame cach element of the plate experiences a pseudo farce proportional to its mass. These forces have a moment which constitutes the bending moment of the problem. To calculate this moment we noe tat the acceleration of an element ata distance & from the axis a= &B and the moment of the forces exerted by the section between x and / is 152 1.305 we eter f Bes- bomp0?--h From the fundamental equation Py 1 3 93) e1FZ~ Sp tape. os _ Ue ‘The moment of inertia T= f 2 dz= UE ca Note that the neutral surface (ie. the suface which contains lines which are neither stretched nor compressed) isa vertical plane here and zis perpendicular to a 4 Hy. $8007 2 negating sce 0, 0 «= 0 egg an, sop (beg ER\2 2 cy" 0 because y= 0 for x= 0 opBrr he yee d= Thus 20 De ee (@), Consider a ollow cylinder of length J, outer radius r + Ar inner radius , fixed at one ‘end and twisted atthe other by means of a couple of moment N. The angular displacement 4, at a distance {from the fixed end, is proportional to both and W. Consider an element of length de at the twisted end, It is moved by an angle @ a8 shown, A vertical section is also shown and the twisting of the paralelopipe of length ! and area Ar di under the action of the twisting couple can be diseussed by elementary means. If fis the tangential force generated then shearing stess is f/Ar dr and this must equal G0= GR, since 0= “E. Hence, f= Garde?P, ‘The force f has moment fr about the axis and so the total moment is No Garb f den BAP BOG 153 (b) For a solid cylinder we must integrate over r. Thus de 2aPdeG_xcg we Jeeps. 283 ° ae 16 ciety We f 22PMrOC. 2 Go usa oo N using G= 81GPa= 81 x1 = 5x10°?m, dy= 3% 10°? ga 2 = Gp radians, = 3m xx Sd xx 2 Ni Ne op (025 - 81) <1 Nan = 05033 x 10°Nm = 0S kN 1.307 The ma is clearly Nw where Nis the moment of the couple producing the maximum permissible torsion, ¢. Thus power that can be tansmitted by means of a shaft rotating about its axis P= BE Ee = 169 kw 1208 Cowie an een sing of wih dra aia rf Be x The pr oui exes cont + Marco hing we ep inside ee onl in he ‘opposite direction. We have for equilibrium ay Bare -aip 154 1.309 1310 where df is the moment of inertia of the elementary ring, B is the angular acceleration and minus sign is needed because the couple N(r) decreases, with distance vanshing at the outer radius, N(r,) = 0. Now d= xeaee P tus oe Th aN aa” 4 MB (e4—r4, om integration or, Nea ), on integration \ | witha, Tr an ‘We assume that the deformation is wholly due to external load, neglecting the effect of the weight of the rod (see next problem). Then a well known formula says, elastic energy per unit volume 1 1 = ystess xstrin = Foe This giee SBC » 004 fre defomaton cc: When a rod is deformed by its own weight the stress increases a5 one moves up, the stretching force being the weight of the portion below the element considered. The stess on the element dr is Ws px? (I-x)g/xP= pg(l-3) The extension of the element is Ade= dAx= pg(l-x)a/E x Integrating A= 3 p g 17/6 is the extension of a de the wholerod. The elastic energy ofthe clement $ ped-» PEED gee be Integrating ae saree Pe Sari (T) L} 158 1311 The work done to make a loop out of a steel band appears as the clastic energy of the Toop and may be calculated from the same. If the length of the band is the radius of the loop R= ABCD of the loop. The elastic energy of this element can be ealeulated by the same sort of arguments as used to derive the formula for internal bending moment. Consider a fibre ata distance 2 from the neutral surface PQ. This fibre experiences a force p and undergoes Zz an cxtewion ds wee = Zp ve P= = Ade, Tene 2 Zea ie ross sectional area of the fibre, the elastic enemgy associated with itis, 1, (2) 58 (2) rapa ig over all the fibres we get El 2_ Eld@ IRL 82 R For the whole loop this gives, 2 Now tof 2naz- Ae cn somecnnyis — LZEME gogus ‘When the rod is twisted through an angle 0, a couple ar 50 appears to resist this. Work done in ewisting the rod by an angle @ & NO= then Jodo 2S 2-75 om puting the values. aPdre ge T oe 1.313 The energy between radii r and r + dr is, By differentiation, xP dr Ge _ 16a? Its density is nee ye 1.314 The energy density is a5 usual 1/2stress x strain. Stress is the pressure pg h. Strain is x p gh by defination of f. Thus w= 1g pet? = 2951/n* on pating the aes. 156 17 1315 1316 HYDRODYNAMICS Between 1 and 2 Hid partices ae in neatly circular motion and therefore have centripetal acceleration The fore for this accleration, like for any other station in an idea! Dud, tam only come from the preature variation along tbe ine joining 1 and 2. This requlres that pressure at 1 should be greater than the pressure at 2 Le Der: so tht the Ouid particles can have required acceleration. If there is no tupbulence, the motion canbe taken as iottional. Then by considering G aro along the circuit shown we infer that er (The ponion of the circuit near 1 and 2 are Streamlines while the other two arms are at right angle to steamlins) In an incompressible liquid we also have div 7°= 0 By electrostatic analogy we then find that the density of streamlines is proportional to the velocity at that pint From the conservation of mass ¥45, = 5, a But S, < S, as shown in the figure ofthe problem, therefore ns [As every streamline is horizontal between 1 & 2, Bemoull’s theorem becomes 1 i P+} pH = constant, which gives P< PaaS % > Ye As the difference in height of the water column is Af, therefore PrP, = shh @ From Bemoul’s theorem between points 1 and 2 of a streamline 1 1 oe pithot=ns ted or, pr-r= k 00h - > or pgAh= ; pF) (3) (using Eq, 2) using (1) in 3), we get vs, zeae VES : Hence the sought volume of water Mowing per see 2 2 m5 = 5,5, VV eA sg 1317 1318 1319 = EHV - VGH 187 Applying Bemoulli’s theorem for the point A and B, Loy Pam Pat ze as, v 1 or, devs pu-pam Aiea. Ah ‘Thus, rate of flow of gas, Q= Sv= 5 ‘The gas flows over the tube past it at B. But at A the gas becomes stationary as the gas will move into the tube Which already contains gas. Inapjig Bemosl hoe we hos emanate const ang 4 streamline. In the present case, we are really applying Bernoulli's theorem somewhat indirectly. The streamline at A is not the streamline at B. Nevertheless the result is correct. To be convinced of this, we need only apply Bemnoull’s theorem to the streamline that oes through A by comparing th situation at A with that above B on the same level. In steady conditions, this agrees with the result derived because there cannot be a transverse pressure differenti Since, the density of water is greater than that of kerosene oil, it will collect at the bottom. Now, pressure due to water level equals hy p,g and pressure due to kerosene oil level ‘equals hy pz g. So, net pressure becomes hp, 8+, P28 From Bernoulli’s theorem, this pressure energy will be converted into kinetic energy while flowing through the whole A. 1 ie ypethames do? Let, H be the total height of water column and the hole is made at a height / from the bottom, ‘Then from Bernoulli’s theorem fpr= Moe or, v= ViE= I) 2g, which is directed horizontally. For the horizontal range, J v £ 168 1320 1321 wich ike he ae 25m. Lethe velo of te water je aah orice bev, ten spying emo tere, Lt moget Jov= hoes hort « va WP o2gh w Here the pressure term on both sides is the same and equal to atmospheric pressure. (In the problem book Fig. should be more clear) Now, if it rises upto a height 5 then at tis height, whole ofits kinetic-energy will be converted into potential energy. So, fev? ph or he x i: = Fo hy = 20em,[osing Ba. (0) Winter flows through the small clearance into the orifice. Let d be the clearance. Then from the equation of continuity (2nRd) vy = (2nd v= (2nRd)¥y or Re vrs mR ® where v,, vp and v are respettively the award radial velocities of the Quid at 1, 2 and 3. ‘Now by Bemoulli’s theorem just before 2 and jjust after it in the clearance 1 Pot hpem prt Sous @= Applying the same theorem at 3 and 1 we find === that this also equals 1 1 P+ xP pot Pye @) (since the pressure inthe orifice i py) From Eqs. (2) and (3) we also hence v= V 26h @ = 70 + hos ( = @ ) [Using (1) and (4) 150 AR Let the force acting on the piston be F and the length of the cylinder be 1. 1323 1324 Then, work done = FI ® Applying Bernoulli's theorem for points Aand B.p= 3 ps? whee piste density and_v isthe velocity at point B. Now, force on the piston, Fx ph= Sova ) Where A isthe cross section area of piston Also, discharge through the orifice during time interval ¢= Sve and this is equal to the volume of the cylinder, Le., v Vasu or vee @ From Eq. (1), @) and (8) work done 4 a = bovis? (@s Al= ¥) Let at any moment of time, water level in the vessel be Hf then speed of flow of water through the orifice, at that moment will be ve V2gH ® In the time interval de, the volume of water ejected through orifice, v= svde @ On the other band, the volume of water in the vessel at time ¢ equals VesH Differentiating (3) with respect to time, a a TSG Wa SAH ® Eqs. (2) and (4) SdH= svdt or dew So V2gi from (2) Inet, fe walt s Thus, = iyz In a rotating frame (with constant angular velocity) the Eulerian equation is Fos oxs 206" 7p - Tp + pis 2067 x} + paki p In the frame of rotating tube the liquid in the “column is practically saic because the orifice is sufficiently small, Thus the Eulerian Eq. in projection form along F” (which is 160 1.325 1.326 the position vector of an arbitrary iquid element of lenth dr relative 10 the rotation axis) reduces to 0 ade 2 “Ps patr=0 @ <—/—~ 6 op = pair VEEP 0, je- uf rir on dl ‘Thus PO = Pot ea ta = d-hy] a) Hence the pressure at the end B — the orifice ie. Pid = po + 25 2th - @ and outside of went po Spat (21h-¥) = pp + E9o? (whee te sought eocy) So, ‘The Euler’s equation is p -F (p+ p.g2), where z is vertically upwards. Now @) But er ® ‘and as the motion is otto! Cul 7 0 So from (1) and @) oe Foren) or, Hes gor" +oee) - 0 Hence p+kpv+pee = constant. “Let the velocity of water, lowing through A be v, and that through B be vg, then discharging rate through A= Q,~ Sv, and similarly through B= S vy. Now, force of reaction at A, rat PQa%a= PSM% 1327 1338 161 Hence, the net force, FepSi-D as Rthh Applying Bernoulli's theorem to the liquid flowing ovt of A we get 1 Pot Pah = p+ 5 PH and similarly at B 1 Po + PB he AM) = py + 5 org Hence Oh oF = dhpg F = 2pg5Ah = 0-S0N Thus Consider an element of height dy at a distance y from the top. The velocity of the fluid coming out of the element is ve Wig ‘The force of reaction dF due to this is dF = pd4v?, as in the previous problem, = p(bdy)2ey + Ineptng Feo foe = pgb [iP -(h- I ]= pgbl2h-D (The slit runs from a depth fT to a depth h from the top.) Let the velocity of water flowing through the tube at a certain instant of time be u then ue 2 xP From impulse momentum theorem, for the stream of water striking the ube comer, in x-direction in the time interval dy F,di= -pQudt ot F, and similarly, F,= pQu Therefore, the force exerted onthe water stream. bythe tbe, Fe ~ Quit eQuj According to thied law, the reaction fo the tube's wall by the stream equals (-F) = pQui-pQuj Heence, the sought moment of force about 0 becomes Na 1(-x (Qu where Q is the rate of flow of water and x 7*is the cross section area of the tube. ~pQu and 162 1.329 Suppose the radius at A is R and it decreases uniformaly to r at B where S = xR? and 5 = a7, Assume also thatthe semi vetical ange at Oi a, Then So yore hr @-Ly where y isthe radius at the point P distant x from the vertex O. Suppose the velocity with which the tguid lows out is V at A, vat Band w at P Then by the equation of continuity HRV = mPy = mu The velocity v of eflx is given by v= Veh and Bernoulli's theorem gives 1 1 Ppt 3 = Pot 3 0 where p, is the pressure at P and py is the atmospheric pressure which isthe pressure just outside of B. The force on the nozzle tending to pull it out is then F=f 5-00) sind 2ayds ‘We have subtracted py which is the force due to atmosphenic pressure the factor sin @ gives horizontal component of the force and ds is the length of the element of nozzle surface, ds = de see 0 and A 8B U a ool tan = Thus = pgh (S~5)'/S = 602N on putting the values. Note : If we ty to calculate F from the momentum change of the liquid lowing out w: will be wrong, even as regards the sign of the force. ‘There is of course the effect of pressure at S and s but quantitative derivation of F fron Newton's law is difficult. 163 1.390 The Euler's equation isp ©= Fp in the space fixed frame where F= - pai” downward. We assume incompressible Auid so p i constant. Then j= - V(pgz) where z is the height vertically upwards from some fixed origin. We go to rotating frame where the equation becomes oe Od the additional terms on the right ae the well known coriolis and centrifugal fores. Inthe frame rotating with te liquid 9” = 0 so Fpsese-pourr) =o =F p+ pg2)+pu*rt2p@" xa) or P+ p82-S parr constant On the free surface p= constant, thus 22 SP + const Itwe choose he origin at point r= 0 (Lethe ans) of he fe surface then “conta” = 0 and 2+ SEP (the paraboloid of revolution) ‘At the bottom z= constant So p= $9.0*F + constant If p= pp on the axis atthe bottom, then P= Pot zp07F, 1331 When the dis rotates the fuld in contact with, corotates but the fd in contact with the walls ofthe cavity does notrotat. A velocity gradients then st up leading to viscous forces. ‘Ata distance r from the axis the linear velocity is or so there is @ velocity gradient h radial width is dr is both in the upper and lower clearance. The corresponding force on the element whose 2dr S (fom the fommular F= 1A 2) ‘The torque due to this force is ‘and the net torque considering both the upper and lower clearance is af meray = xRon/h ‘So power developed is Px atsn/h~= 9-05 W (on puting the values). (As instructed end effets ie. rotation of fuid in the clearance r> R has been neglected) 164 1398 Let cone cor nde of ade rand tikes ten fr of tno ens fc is ements aye, P= Zr! ‘This force must be constant from layer to layer so that steady motion may be possible. @ anim fe _ Q) psteree Fie ant Finges 2xIm% From (2) by (3) we get, Inr/Ry “Oink Ry Note : The force F is supplied by the agency which tries to carry the inner cylinder with velocity vp 1.333 (a) Let us consider an elemental cylinder of radius r and thickness dr then from Newton's formula do pdo Fe 2xrinrSte nin? 2 and moment ofthis force acing on the element, de do No 2nP nftre 22h oe trindae © {AS in the previous problem Nis constant when conditions are steady Intestin, ann faow wf % @ Putting @ 168 From (3) and (4), oro HR 11) onal 7] (®) From Ea. (4), ye Ne anno, aa RR 1384 (a) Let dV be the volume flowing per second though the eylingvca shel of thickness ten, 2 ave -axrarn[i-Z)- and the total volume, ream | (0) Let, dE be the kinetic energy, within the above cylindrical shell, Then ar= dm)? = POurlarp? ~Lorinrari(-2}- sion 2 @ Hence, total energy of the fluid, By) 6 seaweed (© Here frictional force is the shearing force on the tube, exerted by the fluid, which dy equals ~4s ( 2 lane pe Gives, v=(t-B) 50, Fe -t0 And at rok, Ba 200 ar 166 Then vn ace ea F = -n nan 2 -2ntn(-23)- anny! (© Ting eine hl of tes dr and air visosfre Fa -nexn%, Let Ap be the pressure difference, then net force on the element = dp x7? +2 Ir Bu, since the flow is steady, F,.= 0 ~2ainr -2a1 af or ope ang a 1.335 The loss of pressure head in traveling a distance 1is seen from the middle section to be fig= hiy= 10 em, Since fy fiy= fy in our problem and hy hy = 1S m= +hy- hy, We see that pressure head of 5 cm remains incompensated and must be converted into kinetic energy, the liquid flowing out. Thus Thus va V 2gAh «1 m/s 1.336 We know that, Reynold’s number (R,) is defined as, R= pv/n, where v is the velocity is the characteristic length and 1 the coefficient of viscosity. In the case of circular cross section the chracterstic length is the diameter of cross-section d, and v is taken as average velocity of flow of liquid. Now, R,, (Reynoi’s number at x; fom the pipe end) = a where v; isthe velocity at distance x, From equation of continuity, Ay ¥; = Ay¥2 es nrlye ave oF divine davon 78 (as x-x,= Ax) ‘Thus qin thes 1.337 We know that Reynold’s number for turbulent flow is greater than that on laminar flow: Zon Dew Now, (Rays Me FELT ana (Ryn BERNE 1338 1.339 167 But, (RD, = Rds o v= SMe am" Part = Svs on putting the values. ¥ Pod is wi We have R= “TPS and vis given by 6anrv= EP o-pdE (p= density of lead, py = density of glycerine.) ve RO-Er= Bao-edet 1 2 Tou 1. oo-pdend 8 37 pt PEP and d= [97°/p(0~ p01”? = 52mm on puting the values. dv mG, mg-6anr o* Sant, g Seiya gba SAE e eB a halivn get on dtm ge! oe vee eke or ve Ex Ce™ (where Cis const) Since v= 0 for t= 0, 0= £+c So cn -E Thus ve Ba-e™) The steady state velocity is £. v difers fom & by m where « cE Lan We have neglected buoyancy in olive oil. 168 1.8 1340 1.341 RELATIVISTIC MECHANICS From the formula for length contraction (.-uVi-Z baw 2 So, 1-5= O-ny or v= eva Wy (@) In the frame in which the triangle is at rest the space coordinates of the vertices are om, (8.4 S$ frame the corresponding coordinates at time fare 4:00 0,058: SSVI +#'5.0) and c(i 5 w-$.0) ‘The perimeter P is then 18}, a mene a ese ine In he moving 1a =a. VaF) (b) The coordinates in the first frame are shown at time 1. The coordinates in the moving frame are, Pe oera(fa Peg B a a (3) Bo) A 4 ©00) © @o0) iC A(t, 00),B (MIF ow, ‘The perimeter P is then Po aVinB +S [1-3] x 20 a(Vinpr +V4-B*) here B } (VE + w 0,0) v In the rest frame, the coordinates of the ends of the rod in terms of proper length fy A: (0,00) B : (Ip £050, fa sity, 0) at time £ In the laboratory frame the coordinates at time 1! are 1:60,0038:post.Vi-P +t 1303 1344 1345 160 Therefore we can write, 1008 8,= 1p cosy V1 ~ B* and IsinO= iy sin, 2. (42 (90s? 8+ - B*) sin” 8) B Hence 12 wf =e | oe YEE 6 ; eB A In the frame K in which the cone is at rest the coordinates of A are (0,0,0) and of B are (h,htan 8, 0). In the frame K’, which is moving with velocity v along the axis of the cone, the coordinates of A and B at time fare As(-w, 00), B: (I= BF ~w,htan 0,0) Thus the taper angle in the frame K" is ano 228 (222) ‘and the lateral surface area is, Sm xh? sec0' tand! - Se ars TEES «sv Preah Here S,= 2H? sec tan0 is the lateral surface area in the rest frame and Wo AVISBT, Be we. Because of tine distin, + moving clock read ess tne. We wie, to dte VIF, B= Thus, In the frame K the length J of the rod is related to the time of fight At by Ta var In the reference frame fixed to the rod (frame K’)the proper length fy of the rod is, n by But 170 1.346 1.348 1349 Ts, se a oe NEFF wi The dsunce vedi he bray mea ec & whe te vlcy ote pune. Bat by tie dn p — At= ——— 0 ve = (Afp/aty Vi-vyF a Thus the distance traversed is cbt Vi (Aig (@) Itty is the proper life time of the muon th life time in the moving frame is and hence I= tm we (Che words “hom he avn sd pol arent pt of ny stad telat) In the frame K in which the particles are at rest, their positions are A and B whose ‘coordinates may be taken as, A:(0,0,0),B= (lp, 0,0) In the frame K’ with respect to which K is moving with a velocity v the coordinates of A and B at time ¢ in the moving frame are A= (,00)B (6TH #9,00) B= 2 © Suppose B hits a stationary target in K’ after time # while A hits it ater time fy + At. Then, IVP + vg= veg + ds) So, In the reference frame fixed to the ruler the rod is moving with a velocity v and suffers Lorentz contraction. If ly is the proper length of the rod, its measured length will be Ar, VIB, B= = 1.351 1382 a In he reference frame Gted wo the rod the rer sues Loeats contention and we must tive Aa VI=BP = by thas y= Vins as, = and 1-f = = or ve cVi-< Bs ae a The coordinates ofthe ends of the rods inthe fame fed to the lef od are shown, The poins B and D colneides when finb Jom y- My OF f= The points A and E coincide when eth 0» c+ Vi-B Ths are n= #(teVi-F) Mh fy £00) (by 90) (ci-¥t.99) or (a4) = 1-pt= (Coto vA-VE,99) . 22 At/ly 2y/At From tis Tea” Teeny In Ko the rest frame of the particles, the events corresponding to the decay of the particles A: (00,0) and (Oj,0,0)= B Tn the reference frame K, the corresponding coordintes are by Lorentz wansformation ra i wr Vi-# Now 4Vi-B = 1 bby Lorentz Fitgerald contraction formula, Thus the time lag of the decay time of B is ae vt wt Ws Fe evi-p O0-B) ea B decays later (B is the forward particle in the direction of motion) 4:(0,0,0,0), o0| (@) Inthe reference frame K with respect to which the rod fs moving with velocity the coordinates of A and B ate Aitaags 905 )0,0 Bitty v(t~fy),0,0 172 Thus Im x4-2p-V (4-4) = lo VI- ts) sok ©) 2 h-Vq-g)= leVIE (Gince x4~xg can be either + ly of — fy) Thus v(t,-t,)= oF -vi- _™ i 4 ee Wr 1.383. At the instant the picture is 2 the coordintes of A, B,A', Bin the rest frame of AB A:(0,0,0,0) Al B B:(0, 0,0) =o > 4 B’: (0, 0,0, 0) ——=o A B 4°: 0,-4V1=77 0,0) In this frame the coordinates of B' at other times are B' : (f,v, 0,0). So BY is opposite to Se eee ae oer eae hy 4. ‘Similarly in the rest frame of A,B, te coordinates of A at other times are vh-avicE nad As cppnin Wa wine te 8 Vi ‘The corresponding time in the frame in which A’, B' are at rest is PB)= 1.385 1.356 173 So at time 10, = & Ifx>0F <0, if <0, £'>0 and we get the diagram given below “in terms of the K-clock". K9QOO8S8O « DOOOODOOD ‘The situation in terms of the K’ clock is reversed. ‘Suppose x () is the locus of points in the frame K at which the readings of the clocks of both reference system are permanently identical, then by Lorentz transformation “bea Oe Vive se sitaining «(9 © (1-Vi-¥ = Let B= tanh, 0< O<@, Then coshO-1 4 /eoshO=T oo, ov sink” °V coshov ~ hy (4an A is a monotonically increasing function of 8) ‘We can take the coordinates of the two events to be 4: (0,0,0,0) B: (As, a,0,0) Fer we a te sb A scam we tae In the moving frame the coordinates of A and B become 4:0.0,0,0.8:[ (4 } vo-van,0.] where y= Since wie lw we must have Ar’ > 174 1.387 (a) The four-dimensional interval between A and B (assuming Ay = Az 0) is : SF -3 = 16 units ‘Therefore the time interval between these two events in the reference frame in which the events occurred at the same place is €léy=€,)" Vib = 4m a = tents (0) The four dimensional interval eteen 2 and Gssming y= fe 0) F-S = -16 . ‘So the distance between the two events in the frame 1 [TA in which they are simultaneous is 4 units = 4m. O04 234 567 1388 By the veacy on oma woV Vi-Vv77e7 pokaasy and 1.359 (8) By definition the velocity of apporach is dx, dey Vagroun Gen GER ed tM in the reference frame K . (b) The relative velocity is obtained by the transformation law 1.360 The velocity of one of the rods in the reference frame fixed to the other rod is ye vty vie 5 ‘The length of the moving rod in this frame is [ate fn hy aepy aie 1.361 The approach velocity is defined by 7 ae Tae E-Be y-VE x 1.362 1.364 178 On the other hand, the relative velocity can be obtained by using the velocity addition formula and has the components ‘The components of the velocity of the unstable particle in the frame K are [ove] 0 the velocity relative to K is # # Vey? La 2 ‘The life time in this frame dilates to we ee v | V3-3-5 and the distance traversed is ps WF VATE vi - Ve Vi-v Se In the frame K’ the components of the velocity of the particle are —v Hence, ung’ = 2 —¥88 8 VaT ve 6 In K’ the coordinates of A and B are * A:(6,0,~v'f,0),B:(@,1-v',0) ‘After performing Lorentz transformation tothe frame K we get vi 2 se yVeyse y(lV) youl yaad z=0 2-0 Asteyt Bite y(t + By tasting ¢—> =, we can write the coordinates of B as mye 176 Thus av Hence Vy B ' In K the velocities at time ¢ and ¢+ dt are respectively v and v + wde along x ~ axis which is parallel to the vector V. I the frame K’ moving with velocity V'with respect to K, the velocities are respectively, eV iy vewdi“¥ 1 ese ‘The latter velocity is written as ‘Also by Lorentz wansformation ara Ma? | y A-wie Vi-We Vive “Thus the acceleration in the K” frame is aw ae (®) In the K frame the velocities of the particle at the time and 1+ di are repectively (0, v,0) and (0, v + wet, 0) where Wis along x-axis. In the K’ frame the velocities are (-YevizP7e 0) and (-¥. + wd VI=V7E 0) respectively 77 ‘Thus the acceleration MIT Dag te pe we We have used a’ = —— 4 vi-We 1.366 In the instantaneous rest frame v= V and So, i constunby samp Thus ign ge ee Vip rmegune wee ein 2= (Vio 1.367 The boost time ty in the reference frame fixed to the rocket is related to the time + elapsed fon the earth by ; iol 1a For Bete - * my” VEC= BY)” VER 1.369 We define the density p in the frame K in such a way that p de dy dz is the rest mass dy ofthe element. That is p de dy dz™ py dtp djy diy , where py is the proper density 4p, dyy, dg are the dimensions of the element in te rest fame Ky Now y= dy, dem diy, dem depy/, 7 178 if We frame K is moving with velocity, v relative to the frame Ky. Thus Defining m by p= po(t +n) 1, n@+n) te - n@+w (enh +n) We get 141 Vier . Yneen (+n)? i+ 1.370 We have ~ seb be 1.371 By definition of n, or ve eVi-d = Vand a7 ava 1.372 The work done is equal to change in kinetic energy which is Classically ie. in nonrelativistic mechanics, the change in kinetic energy is A mye? (08? -(06)?) = 4m 2028 = 014m) Relatvisically it is, my moe me _ moe Vi-c8F Vi-@oF 06 08 = 0-416 my e2= O42 mc mg? (1-666 ~ 1-250) 1374 1375 1376 176 Relavistially So Thus , TT ,, Ba-Bas 37 But Ctasically, y= WE, s0 sea mssicaly, Bae VO fa ame * T 4 Hence i wade the velocity B is given by the classical formula with an error less than €. From the formula ame my , vz vi-vie ve wefind E%= cpt emict ot (mete T= pte mc! LV ame+1) Let the total force exerted by the beam on the target surface be F and the power liberated there be P. Then, using the result of the previous problem we see P= Np= AVE 42m) = L VT T+ Im since 1= Ne,N being the number of particle striking the target pet second. Also, mye Vi-vie ‘These will be, respectively, equal to the pressure and power developed per unit area of the target i€ J is current density. or TQme+D= ep? ic p PaN 180 1377 In i frame fixed to the sphere :- The momentum transferred to the eastically scatterred particle is ‘The density of the moving element is, from 1.369, » ‘and the momentum transferred per unit time per unit area is 2m etn ys ame : Vie Vie Oe eer eer P= the pressure = ‘The momentum transferred is and the pressure is 27%. nyo 20 e te 1.378 The equation of motion is using v= 0 for r= 0 2 en ) Beate tmek & ”” Vmgore rr 181 1.381 By definition, E=m where dd? a=? iste invariant interval (dy = d= 0) = Jpn omy egy EEN, BER MELS Ts Par oma ge omot vow “8 eon Bm mo E—2 my 1.382 For a photon moving in the x direction = Py» By™ Pe™ 0, In the moving frame, Note that ee 1.383 As before 182 1.384 1.385 1.386 Sainty a medtine me Then Bee BoP (et 2 ¢4 (Cd - dr? ~ dy*-d?*) a mget Ca ae = dy de) at mic! is invariant (©) & @) In the CM frame, the total momentum is zero, Thus Vu VET rm) T ce” E+E,” T+2mc* T+2my where we have wsed the result of probiem (1.375) Then 1 1 af[TrIme Wwe ve T+2mgce ‘Total energy in the CM frame is ano JTL Ree Vim AT FIM =F Sar” mae ome Zmy cr (T+ Img) = T+ 2m? So Tt. meV -| Asso 2VEf + mge! = VingF Time), 427 = Ime, ot F Mc=VE-op Veg Fe TFT Ome + T) = Virgo mye + T= eVimCamge™ 1) Also p= VI(F+ImA), ve 2. e Let T = kinetic energy of a proton striking another stationary particle of the same rest ‘mass. Then, combined kinetic energy in the CM frame «mel es). 1387 1.388 183 We have EytEytEy= mA PPh 0 Hee (me-E Fe PT= E+E GHB A The LHS. = (mbct-£,)'- Apt (nbs mic! -2m cE, ‘The RHLS. is an invariant. We can evaluate it in any frame. Choose the CM frame of the particles 2 and 3. In this frame RH.S. = (E',+ E's)" = (m, 4 m,)c* Thus (mg+ mi) ct 2m, 2B, = (my +m.) ct enim (mem, of 2myetEy« {mgmt (m, +m] 4, or py Tenia The velocity of ejected gases is u realtive to the rocket. In an earth centred frame it is “e in the direction of the rocket. The momentum conservation equation then reads (mdm) (v + dv) + = or dy -("=1—v)dm= 0 1-2 é Here ~ dm is the mass of the ejected gases. so PART TWO THERMODYNAMICS AND MOLECULAR PHYSICS 2.1 EQUATION OF THE GAS STATE + PROCESSES 2.1 Let m, and m, be the masses of the gas in the vessel before and after the gas is released+ Hence mass of the gas released, ‘Am = m,-m, Now from ideal gas equation PV= m ET, and pyV= m ET, as V and T are same before and ler the release of the gas. 80, (P,-P2) V= (m,-, m) Fe Ty= ane @-P)VM _ apvM o am Scope. ee ® Westen p= pT we Hon 2 @ (where py=_ standard atmospheric pressure and Ty = 273 K) From Eqs. (1) and (2) we get ama VRP 13%30%° 2.2 Let m, be the mass of the gas enclosed. Then, PiV=v,RT, ‘When heated, some gas, passes into the evacuated vessel tll préssure difference becomes Ap. Let p'; and p’; be the pressure on the two sides of the valve. Then PiV= ¥,RT, and P2V=VRT,= (v,-V)RTy 188 bu, yas Pat bp) ~P2- AP » 1 (at *, pe = (222 99|= conan 2.3 Let the mixture contain v, and vp moles of Hy and H, respectively, If molecular weights of H and H, are M, and Mz, then respective massec inthe mixture are equal to m= ¥,M, and m= vpMz ‘Therefore, for the total mass of the mixture we get, me m,4m, ot m= ¥iM,+¥yMy ® ‘Also if vis the total number of moles ofthe mixture in the vessels, then we know, ve yey @ Solving (1) and (2) for vy and vp we get, ye Ma=m) ) ma¥ My, Ae “Theret mee My SAE sn me MyM refore, we get my= My “pry and m= Maye ap mM, (My~m) o) my” My (m= Mi) One can also express the above result in terms ofthe effective molecular weight Mf of the mixture, defined 25, m_ RT vo My -My-M_1-M/M, mm” My M=M,” M/M,=1 Thus, Using the data and table, we get M= 30g and, “= 050 my 24 We know, for the mixture, N, and CO; (being regarded as ideal gases, their mixture too behaves like an ideal gas) PV=VRT, so pyV= vRT where, v is the total number of moles of the gases (mixture) present and V is the volume of the vessel. If'v, and v, are number of moles of N, and CO, respectively present in the mixture, then ve yey Now number of moles of Nz and CO, is, by de where, m, is the mass of N (Moleculer weight of CO, (Molecular weight = M,) in the mixture. ‘Therefore density of the mixture is given by m4 My en” CRIP) = Po Mita Po (my + my) My My RT vy+vy ” RT (HM +m, M,) = 15 kg/m? on substitution 25 (a) The mixture contains v, ,v, and v, moles of Oy, N, and CO, respectively. Then the total number of moles of the mixture ve vtytys We know, ideal gas equation for the mixture yRT pV=vRT ot p=“ tyr yyaT Go po SATYAEYDAT 6 am on substation (b) Mass of oxygen (O,) present in the mixture : m= ¥, My ty v2Mz fe (CO,) present in the mixture : my= v3 My Mass of nitrogen (N;) present in the mixture : Mass of carbon i So, mass of the mixture m= my+m;+ms= VM, +¥2M,+¥5My mass of the mixture Total number of moles Moleculer mass of the mixture : Mf = aM + vay +5 My = 367 g/mol. on substitution 26 27 187 Let p, and p, be the pressure in the upper and lower part of the cylinder respectively at temperature Ty At the equilibrium position for the piston : pyS+me~ poS ot p+ 2 pa (mi the mass ofthe piston) aT, ut p= £7 (were Vp isthe inal volume of the lower pat) a ar, RY, RI, Se me. Ate ome. S00; 1) So, ms" % 37% (0 a 9 Let T' be the sought temperature and at this temperature the volume of the lower part becomes V, then according to the problem the volume of the upper part becomes 7'V Hence, BFE. ‘ ® From (1) and @) Ma(.3) a ca)” Vo n * Vy (! 1 ‘Ass he oa volume must be constant, Votem= Witen’) of, ve tw +n) Putting the value of Vin Eq. (3), we get Let p; be the density after the frst stroke. The the mass remains constant = + pe ee Vp=(V+d oe» % Mi" Tan Similarly, if pis the density after second stroke (_V_\, .(_Vv_) Fan (tam Ver (V+AV)p2 0% pam In this way after nth stroke. v Pa" (Wav) PO Since pressure o& density, 188 28 29 r tes (eg) meme em cmt , Pa 1 It is required by 2 tobe 4 aired by Tt 5 Liv “ a” (7 + av) Hence m RT From the idea gus equation p= AZ a) In case of continuous ejection, if (my ) corresponds to mass of gas in the vessel at time 1, then my is the mass at time ¢+ Af, where Ay, is the 1¢ in which volume v of the gas thas come out. The rate of evacuation is therefore i.e. Un he int 8-0, we co tat From @) a @) 2. a inegning f 2- “grating, P ‘Thus pepe Let p be the instantaneous density, then instantaneous mass = V,, In a short interval d the volume is increased by Cat. So, . Vp= W+Ca)(p-+dp) (because mass ssmains constant in a short interval dt) 210 2a 189 s0, a. Since pressure a density 2= res yo h or f-*- ? Mig te igh or ta Eine Lint 10min “The physical system consists of one mole of gas confined in the smooth vertical tube. Let 1m, and m be the masses of upper and lower pistons and S, and 5; are their respective For the lower piston pS, +m,g= pS, +T, oy, T= @-p)S,+mE ® Simitaly forthe upper piston PoS,+T#mg~ D5, or, T= @-p)S\-m8 ® From (1) and (2) © Po) -S)= (m+) o —(P=p)AS= mg 50, p= The py= constan P= AS + Po™ constant From the gas law, pV= vRT PAV= RAT (because p is constant) +78) as1= a foe B8)gor- nar Hence, ATs EGpAS+mg) I= 09K (@s, Ve R7/p for one mole of g35) 1 Thus, REPRO? ~ Re Veo -P « 2p Foray LP) see 20 imp 190 22 23 wick yi, p= 2 © te ta hen Vn 3GVE ©) pm mePY = pe br? so Bem in he, and Te fen ® For Tay the conn is $= 0, which yields 3 Hence using this value ofp in Eq, (1), we get F (as, V= RT/p for one mole of gas) $0, p= Varrq-T) ” ror aus B+ 0,0 ses T=2T, @ From (1) and (2), we get, Pain VER2q(2Ty- 7)" = 2RVETS Consider a thin layer at a height h and thickness dh. Let p and dp + p be the pressure on the two sides of the layer. The mass of the layer is Sdhp. Equating vertical downward force to the upward force acting on the layer. Sdh p g+ (p+ dp)S= pS so. $a -oe © Lb Jordes Bu, p= Lar, wenive dp» Mar, tht sdhg o, - Rare , Rare eds | aT gM, G2 34K em ‘That means, temperature of air drops by 34°C at a beight of 1 km above bottom. So, 2d 2s 216 ave, Bm ~ pg (See Weave, $= pe (ee 219 ® Das fom po CF (oh Cn cnt ong ® We tve tom sas tow pe pT, worn @) ae Moat Cee pT o TH Eee aM 2 Thus, LM cian 8 a (m=) ® (aes oe But, dh” dp dp dh Me (yay? pgy~ MED 8, Fe ewe ac a= ee M We have, dp= ~ p gdh and from gas law p= 2 P a rs @, Me T™ we. TE an Integrating, we get ’ . 2. Met, or, in 2a ME, ” (where pp is the pressure at the surface of the Earth.) pe pera, [Under standard condition, py= 1 atm, T= 273K ~ 281941 50078141273 © 9.5 atm, ~2 x98 4(-SHOYEAT «> stm) Pressure at a height of $ atm = 1xe Pressure in a mine at a depth of 5 km = 1xe We have dp ~pedh but tom ens lw p= LRT, ‘Thus p= SRT a const temperature do, EM par

Potrebbero piacerti anche